You are on page 1of 62

VISIONIAS

Test Booklet Series

TEST BOOKLET

GENERAL STUDIES (P) 2023 – Test – 3816


C
Time Allowed: Two Hours Maximum Marks: 200

INSTRUCTIONS

1. IMMEDIATELY AFTER THE COMMENCEMENT OF THE EXAMINATION, YOU SHOULD CHECK THAT THIS BOOKLET
DOES NOT HAVE ANY UNPRINTED OR TURN OR MISSING PAGES OR ITEMS, ETC. IF SO, GET IT REPLACED BY A
COMPLETE TEST BOOKLET.

2. ENCODE CLEARLY THE TEST BOOKLET SERIES A, B, C OR D AS THE CASE MAY BE IN THE APPROPRIATE PLACE IN
THE ANSWER SHEET.

3. You have to enter your Roll Number on the Test Booklet in the Box
provided alongside. Do NOT write anything else on the Test Booklet.

4. This Test Booklet contains 100 items (Questions). Each item is printed in English. Each item comprises four
responses (answers). You will select the response which you want to mark on the Answer Sheet. In case you
feel that there is more than one correct response with you consider the best. In any case, choose ONLY ONE
response for each item.

5. You have to mark all your responses ONLY on the separate Answer Sheet provided. See direction in the

O
answers sheet.

C
6. All items carry equal marks. Attempt all items. Your total marks will depend only on the number of correct
responses marked by you in the answer sheet. For every incorrect response 1/3rdof the allotted marks will be
S.
deducted.
TE

7. Before you proceed to mark in the Answer sheet the response to various items in the Test booklet, you have to
fill in some particulars in the answer sheets as per instruction sent to you with your Admission Certificate.
O

8. After you have completed filling in all responses on the answer sheet and the examination has concluded, you
FN

ator only the answer sheet. You are permitted to take away with you the Test
should hand over to Invigilator
Booklet.
D

9. Sheet for rough work are appended in the Test Booklet at the end.
.P
W

DO NOT OPEN THIS BOOKLET UNTIL YOU ARE ASKED TO DO SO


W

1 www.visionias.in ©Vision IAS


W
.
1. With reference to 'expunction' or removal of 3. Consider the following statements regarding
certain portions of parliamentary the DNA and RNA vaccines:
1. Unlike RNA vaccine, DNA vaccines
proceedings in India, consider the following
have potential to integrate into host cell
statements: genome.
1. The Speaker of the Lok Sabha has the 2. RNA vaccine uses a copy of a natural
discretion to expunge the words or usage chemical called messenger RNA
(mRNA) to produce an immune
from the proceedings of the house.
response in humans.
2. The expunged portions of the Which of the statements given above is/are
proceedings cease to exist in the records correct?
of Parliament, but they can be reported (a) 1 only
(b) 2 only
by media houses.
(c) Both 1 and 2
3. The expunction is an Indian innovation (d) Neither 1 nor 2
and is not mentioned under the Rules of
Procedure and Conduct of Business. 4. Recently seen in the news ' Kappaphycus' is
a/an:
Which of the statements given above is/are
(a) rare species of bat found in Meghalaya.
correct? (b) new species of ginger discovered in
(a) 1 only Andaman and Nicobar Island.
(b) 2 only (c) invasive seaweed.
(d) species of ant discovered from Western
(c) 1 and 3 only
Ghat.
(d) 2 and 3 only
5. With reference to the provisions of the
2. Recently, INS Trikand participated in Sea Protection of Children from Sexual Offences
(POCSO) Act, 2012 and the POCSO
Phase-I of the International Maritime
(Amendment) Act, 2019 in India, consider
Exercise/Cutlass Express 2023 (IMX/CE-23) the following statements:
held in the Gulf. Which of the following are 1. It was enacted as a consequence to

O
the participating countries in the Cutlass India’s ratification of the UN

C
Convention on the Rights of the Child in
Express 2023?
1992.
S.
1. Saudi Arabia 2. It allows reporting of sexual offences up
TE

2. USA to 3 years from the date of the


3. Japan commission of a sexual offence.
3. It provides for a punishment of a
O

4. Iran
minimum of 5 years for using a Child
FN

Select the correct answer using the code for pornographic purposes.
given below. Which of the statements given above are
D

(a) 1 and 2 only correct?


.P

(a) 1 and 2 only


(b) 1, 2 and 3 only
(b) 2 and 3 only
W

(c) 3 and 4 only (c) 1 and 3 only


(d) 1, 2, 3 and 4 (d) 1, 2 and 3
W

2 www.visionias.in ©Vision IAS


W
.
6. Consider the following statements: 9. Consider the following statements:
1. A few organisms that can tolerate and 1. Central Board of Film Certification
thrive in a wide range of temperatures (CBFC) is a statutory body under the
are stenothermal organisms. Ministry of Information and
2. Vast majority of organisms are restricted Broadcasting.
to a narrow range of temperatures are 2. The members of the CBFC are
eurythermal organisms. nominated by the Central Government
3. The levels of thermal tolerance of by drawing people from different walks
different species determine to a large of life for a period of 2 years.
extent their geographical distribution. Which of the statements given above is/are
Which of the statements given above is/are correct?
correct? (a) 1 only
(a) 1 and 2 only (b) 2 only
(b) 2 only (c) Both 1 and 2
(c) 1 and 3 only (d) Neither 1 nor 2
(d) 3 only
10. With reference to the Indian Parliament,
7. Consider the following statements with consider the following statements:
reference to Ujjawala Scheme: 1. The percentage of bills referred to
1. Its main objective is to prevent Committees has been steadily declining
trafficking, rescue, rehabilitation and re- since the 14th Lok Sabha.
integration of victims of trafficking for 2. Indian constitution provides a fixed
commercial sexual exploitation. parliamentary calendar.
2. It is central sector scheme, launched by Which of the above statements is/are not
Ministry of Home Affairs. correct?
Which of the statement given above is/are (a) 1 only
correct? (b) 2 only
(a) 1 only (c) Both 1 and 2
(b) 2 only (d) Neither 1 nor 2
(c) Both 1 and 2
(d) Neither 1 nor 2 11. With reference to TiHAN (Technology
Innovation Hub on Autonomous

O
8. Consider the following statements regarding Navigation), consider the following

C
missile technology: statements:
S.
1. Cruise missiles fly within the earth’s 1. It is India’s first autonomous navigation
atmosphere and use jet engine facility for ground and aerial vehicle
TE

technology. testing.
2. Ballistic missiles are powered by rockets 2. It is developed by the Indian Space
O

initially but then they follow an Research Organisation (ISRO) to


unpowered, free-falling trajectory augment the Indian Regional Navigation
FN

towards their target. Satellite System (IRNSS).


Which of the statements given above is/are Which of the statements given above is/are
D

correct? correct?
.P

(a) 1 only (a) 1 only


(b) 2 only (b) 2 only
W

(c) Both 1 and 2 (c) Both 1 and 2


(d) Neither 1 nor 2 (d) Neither 1 nor 2
W

3 www.visionias.in ©Vision IAS


W
.
12. Which of the following statements best 14. Which of the followings are the favourable

describes the iron catastrophe? conditions for hail formation?

(a) It marked the beginning of Earth’s 1. High liquid water content

oxygen-rich atmosphere some 2.5 billion 2. Great vertical extent of the

to 2.3 billion years ago. cumulonimbus cloud

(b) It is the explosive death of a star and 3. Good portion of the cloud layer is below

often result in the star obtaining the freezing

brightness of 100 million Suns for a 4. Low surface temperatures

short time. 5. Strong updraft within the parent

(c) It occurs when the black hole distort the thunderstorm

space around them and suck Select the correct answer using the code

neighbouring matter into them including given below.

stars. (a) 1 and 5 only

(d) It is a geological event, where the earth’s (b) 1, 3 and 4 only

temperature heated to the melting point (c) 2, 3 and 5 only

of iron and leading to the formation of (d) 1, 2, 3 and 5 only

early core.

15. Consider the following statements regarding

13. Consider the following statements regarding the wheat cultivation:

the impact of currency devaluation by a 1. India is the second largest producer and

country: consumer of wheat.

1. It may increase the export 2. World trade in wheat is greater than for

O
competitiveness of the domestic country. all other crops combined.

C
S.
2. It may decrease the debt burden of 3. Heavy soil with good drainage are

foreign-denominated loans when priced table for wheat cultivation under dry
suitable
TE

in the home currency. conditions.


O

Which of the statements given above is/are Which of the statements given above is/are
FN

correct? correct?

(a) 1 only (a) 1, 2 and 3


D

(b) 2 only (b) 1 only


.P

(c) Both 1 and 2 (c) 1 and 3 only


W

(d) Neither 1 nor 2 (d) 2 only


W

4 www.visionias.in ©Vision IAS


W
.
16. In addition to making the Constitution and 18. Consider the following statements regarding
enacting ordinary laws, the Constituent the tsunami:
Assembly performed other functions also. In 1. When tsunami leaves deep waters, its
the context of this, consider the following speed reduces but the change in the total
statements: energy of the tsunami remains constant.
1. It adopted the national flag on August 2. The rate of energy loss of a tsunami
15, 1947. wave is inversely related to its
2. It adopted the national anthem on wavelength.
January 24, 1950. 3. The ‘tsunami ready’ tag is given to the

3. It adopted the national song on January countries by the UNESCO.

24, 1950. Which of the statements given above is/are

4. It elected Dr. Rajendra Prasad as the first correct?


(a) 3 only
President of India.
(b) 1 and 2 only
Which of the statements given above are
(c) 2 only
correct?
(d) 1, 2 and 3
(a) 1 and 4 only
(b) 2 and 3 only
19. The term “Island Chain Strategy” is
(c) 1, 3 and 4 only
sometimes mentioned in the news in the
(d) 2, 3 and 4 only
context of the containment of which of the
following country?
17. Consider the following statements with
(a) China
respect to community health workers and
(b) Indonesia
services:
(c) India
1. Anganwadi workers and Auxiliary
(d) Philippines
Nurse midwives (ANM) act as resource
persons for the training of Accredited
20. Consider the following statements:
Social Health Activist (ASHA) workers.
1. Humayun invited Mir Sayyid Ali and

O
2. Anganwadi services are provided by the Abdus Samad to establish a studio in his
Integrated Child Development Services
C
court and carry out royal paintings.
S.
(ICDS) Programme under the Ministry 2. The first major project undertaken
of Women and Child Development.
TE

during Humayun’s regime was that of


3. The respective state governments illustrating the Hamza Nama.
O

decides the honorarium to be given to 3. Paintings under Akbar’s reign were a


ANMs, ASHA and Anganwadi workers. fusion of Persian and Indian styles.
FN

Which of the statements given above is/are Which of the statements given above are
correct? correct?
D

(a) 1 and 2 only (a) 1 and 2 only


.P

(b) 2 and 3 only (b) 2 and 3 only


W

(c) 2 only (c) 1 and 3 only


(d) 1 and 3 only (d) 1, 2 and 3
W

5 www.visionias.in ©Vision IAS


W
.
21. In the context of planning in India, which of 23. Consider the following statements regarding
the Ways and Means Advances (WMAs) of
the following statements regarding Five
RBI:
Year Plans is not correct? 1. WMAs are not part of the Fiscal
Responsibility and Budget Management
(a) Drought Prone Area Development
Act (FRBM), 2003.
Programme (DPAP) was launched 2. The interest rate for Special Drawing
Facility under WMA is one percentage
during sixth five year plan.
point more than the repo rate.
(b) The Rolling Plan was implemented after 3. There is a state-wise limit for the funds
that can be availed via WMA.
fifth five year plan.
Which of the statements given above is/are
(c) The actual growth of the eighth five year correct?
plan was more than targeted growth. (a) 1 only
(b) 1 and 3 only
(d) Towards Faster & More Inclusive (c) 2 and 3 only
Growth was the objective of eleventh (d) 1, 2 and 3

five year plan. 24. Consider the following statements with


reference to India’s external trade:
1. India’s exports crossed the value of 1
22. Consider the following statements regarding trillion dollars recently.
Hoysala architecture: 2. No south Asian country is among the top
10 export destinations.
1. It is predominantly found in southern
Which of the statements given above is/are
Karnataka. not correct?
(a) 1 only
2. It is made from gneiss - a relatively hard
(b) 2 only
stone. (c) Both 1 and 2
(d) Neither 1 nor 2
3. It is based on a stellate (star-like) ground

O
plan along with a profusion of 25. In the context of the Portuguese in India,

decorative carvings.
C
which of the following statements best
S.
describes the ‘Cartaz System’?
4. Hoysala temples have pyramidal (a) It was a license issued to Portuguese by
TE

the Zamorin of Calicut to setup factories


vimana.
in Kerala.
O

Which of the above statements is/are (b) It was a royal charter granting the
FN

Portuguese East India Company


correct?
important trading facilities in Goa.
(a) 1 and 3 only (c) It was a license granting duty
du free trade
D

of spices and medicines.


.P

(b) 2, 3 and 4 only


(d) It was a naval trade license or pass
(c) 3 and 4 only
W

issued by the Portuguese in Indian


(d) 1, 3 and 4 only Ocean.
W

6 www.visionias.in ©Vision IAS


W
.
26. Consider the following statements regarding 28. Consider the following statements:

Affordable Rental Housing Complexes 1. Longitude is the angular distance of a

(ARHC): point on the earth’s surface, measured in


degrees from the center of the earth
1. The ARHC Scheme was announced
whereas Latitude is an angular distance,
under the Atmanirbhar Bharat Package
measured in degrees along the equator.
during the Covid-19 pandemic.
2. The average distance between two
2. It was devised to provide affordable
latitudes is 111 Kilometers.
rental housing solutions for urban
Which of the statements given above is/are
migrants/poor. correct?
3. The workforce in manufacturing (a) 1 only
industries, the service sector, laborers, (b) 2 only
and students will be the target (c) Both 1 and 2

beneficiary under ARHCs. (d) Neither 1 nor 2

4. It is launched by the Ministry of Labour


29. Which of the following suggested for the
& Employment.
first time that the residuary powers be
Which of the statements given above are
transferred from the Union List to the
correct?
Concurrent List, except for the residuary
(a) 1 and 2 only
power to impose taxes?
(b) 1, 2 and 3 only
(a) Rajamannar committee (1969)
(c) 3 and 4 only (b) National Commission to Review the
(d) 1, 2, 3 and 4 Working of the Constitution (2000)
(c) Punchhi Commission (2007)
27. Which of the following are west-flowing (d) Sarkaria Commission (1983)

O
rivers in India?

1. Luni River 30.


C
Consider the following statements about the
S.
fort of Agra:
2. Tapti River
TE

1. It is made up of red sandstone and it was


3. Kaveri river
Akbar.
built by Emperor Akbar.
4. Periyar River
O

2. It is a UNESCO World Heritage Site.


Select the correct answer using the code
FN

Which of the statements given above is/are


given below.
correct?
D

(a) 1 and 2 only (a) 1 only


.P

(b) 1, 2 and 4 only (b) 2 only


W

(c) 1, 2 and 3 only (c) Both 1 and 2

(d) 3 and 4 only (d) Neither 1 nor 2


W

7 www.visionias.in ©Vision IAS


W
W
W
W
.P
D
FN
O
TE
S.
C
O
W
W
W
.P
D
FN
O
TE
S.
C
O .
W
W
W
.P
D
FN
O
TE
S.
C
O .
.
46. With reference to the Jammu and Kashmir 48. With reference to zero coupon zero principal
Delimitation Commission, consider the instruments, consider the following
statements:
following statements:
1. These are issued by a not-for-profit
1. It was constituted by the Government of organization that is registered with the
India, in the exercise of powers social stock exchange.
conferred by the Delimitation Act, of 2. An entity issuing these securities does
not have to repay interest and the
2002.
principal.
2. The Delimitation Commission was Which of the statements given above is/are
entrusted with the work of delimiting correct?
both the Assembly and Parliamentary (a) 1 only
(b) 2 only
Constituencies on the basis of the 2011
(c) Both 1 and 2
Census. (d) Neither 1 nor 2
3. The Commission did not provide for the
49. In the context of modern Indian history,
reservation of seats for the Scheduled
Sibirams, military-style camps, set up in the
Tribes in the Legislative Assembly of
region of east and west Godavari, Krishna,
J&K. and Guntur districts were commonly set up
Which of the statements given above is/are as
(a) the headquarters of the Swadeshi
correct?
Movement.
(a) 1 and 2 only (b) the headquarters of the Salt Satyagraha.
(b) 2 and 3 only (c) to conduct the meetings of parallel
(c) 1 and 2 only governments.
(d) the storehouse of the ammunition.
(d) 3 only

50. Consider the following statements with


47. In the context of blood circulation in human regard to the Indian Universities Act, of
beings, consider the following statements: 1904:

O
1. It was based on the recommendations of
1. Human heart is separated into the right
the Raleigh Commission.
side and the left side to keep oxygenated
C
2. The government was provided with
S.
and deoxygenated blood from mixing. power to veto the regulations made by
TE

2. The carbon dioxide-rich blood is brought the senates of the universities.


3. It provided for a sanction of five lakh
to the lungs and the oxygenated blood
rupees per annum for five years for the
O

from the lungs is brought to the heart. improvement of higher education and
FN

Which of the statements given above is/are universities.


correct? Which of the statements given above are
D

correct?
(a) 1 only
(a) 1 and 2 only
.P

(b) 2 only (b) 2 and 3 only


W

(c) Both 1 and 2 (c) 1 and 3 only


(d) Neither 1 nor 2 (d) 1, 2 and 3
W

11 www.visionias.in ©Vision IAS


W
.
51. With reference to the President's address to 54. With reference to the Sarus crane, consider
Parliament at the commencement of the first the following statements:
session of each year, consider the following 1. It is the tallest flying bird in the world.
2. It is listed in Schedule IV of the Wildlife
statements:
(Protection) Act 1972.
1. The Constitution of India mentions the
3. It is monogamous however due to
address of the President to the House of
climate change it showing different
People at the commencement of the first behavioural patterns.
session after each general election. Which of the statements given above are
2. Discussion on a Motion of Thanks is correct?
moved by a member of parliament (a) 1 and 2 only
selected by the Lok Sabha speaker. (b) 2 and 3 only
(c) 1 and 3 only
Which of the statements given above is/are
(d) 1, 2 and 3
correct?
(a) 1 only
55. Consider the following statements with
(b) 2 only reference to the economy of the Bahmani
(c) Both 1 and 2 kingdom:
(d) Neither 1 nor 2 1. The title of Malik-ul-Tujjar, which
means 'chief of merchants', was granted
52. With reference to the e-waste generation in to Mahmud Gawan.
India, consider the following statements: 2. The Bahamani Kingdoms were divided
into provinces called tarafs.
1. India is 3rd largest Electronic waste
3. Dabul was a major commercial port
producer in the world.
during the period.
2. Less than 2% of electronic waste Which of the statements given above are
generated in India is recycled through correct?
institutional processes. (a) 1 and 2 only
3. Waste batteries are not covered under (b) 2 and 3 only
the E-waste (management) Rules 2022. (c) 1 and 3 only
Which of the statements given above is/are (d) 1, 2 and 3

correct?

O
56. With reference to the district administration
(a) 1, 2 and 3
in India, consider the following statements:

C
(b) 1 and 2 only 1. The power to create new districts or alter
S.
(c) 3 only or abolish existing districts rests with the
(d) 2 and 3 only State governments.
TE

2. It can either be done through an


53. The term ‘Multi-Party Interim Appeal executive order or by passing a law.
O

Arbitration Arrangement’ is related to: 3. The Centre has no role to play while
changing the name of a district or a
FN

(a) Insolvency and Bankruptcy Code of


railway station.
India
Which of the statements given above is/are
D

(b) United Nations Conference on Trade and correct?


Development
.P

(a) 1 and 2 only


(c) World Trade Organization (b) 2 only
W

(d) International Centre for Settlement of (c) 1 and


and 3 only
Investment Disputes (d) 1, 2 and 3
W

12 www.visionias.in ©Vision IAS


W
.
57. Which one of the following is the only state 60. Consider the following statements regarding

in India to have an exclusive cut-off date for the Office of Chief Labour Commissioner

citizenship mentioned under Citizenship Act, (Central):

1955? 1. It is responsible for the prevention and

(a) Nagaland settlement of industrial disputes through


conciliation/mediation.
(b) Punjab
2. It verifies the membership of Trade
(c) Assam
Unions affiliated to the Central
(d) Tamil Nadu
Organisations of workers.
3. It is responsible for the revision of the
58. In the context of international trade, consider
dearness allowance component of
the following statements regarding Asian
minimum wages in the scheduled
Clearing Union (ACU):
employments under the Minimum
1. It was established in 1974 at the
Wages Act, 1948.
initiative of the United Nations
Which of the statements given above is/are
Economic and Social Commission for correct?
Asia and Pacific (ESCAP). (a) 1 only
2. Majority of transactions under ACU are (b) 1 and 2 only
carried out in the domestic currencies of (c) 2 and 3 only
the member countries. (d) 1, 2 and 3
3. India is not a member of ACU.

Which of the statements given above is/are 61. With reference to Flex Fuel vehicles,

correct? consider the following statements:

O
(a) 1 only 1. Unlike traditional vehicles, flex-fuel

C
(b) 2 only vehicles are capable of operating on a
S.
blend of gasoline or methanol.
(c) 2 and 3 only
2. These vehicles need to have two
TE

(d) 1 and 3 only


separate tanks for two fuels the engine
O

runs on one fuel at a time.


59. Which of the following doctrines are
FN

Which of the statements given above is/are


associated with Jainism?
correct?
(a) Madhyamika
D

(a) 1 only
.P

(b) Anekantavada
(b) 2 only
(c) Sarvastivada
W

(c) Both 1 and 2


(d) Anattavada
(d) Neither 1 nor 2
W

13 www.visionias.in ©Vision IAS


W
.
62. Recently, the Supreme Court had an all- 65. Consider the following statements regarding
woman bench, in the context of this consider the French East India Company:
the following statements:
1. The First French factory was established
1. It is the first time in the history of the
at Calcutta in December 1700.
Supreme court that an all-woman bench
2. Its directors were nominated by the king
was formed.
2. Justice M Fathima Beevi became the of France.
first woman judge to be appointed to the 3. The French East India Company lost its
apex court. Indian presence forever after losing to
Which of the statements given above is/are
the Dutch in the Battle of Chinsura.
correct?
Which of the statements given above is/are
(a) 1 only
(b) 2 only correct?

(c) Both 1 and 2 (a) 1 and 3 only


(d) Neither 1 nor 2 (b) 2 only
(c) 2 and 3 only
63. Which of the following is correct regarding
(d) 1 only
Gross Domestic Product of a country?
(a) Not all productive activity is included in
GDP calculation. 66. Consider the following statements about the
(b) It is used to determine the status of Least Universal Service Obligation Fund (USOF):
Developed Countries by the UN. 1. It ensures that there is universal non-
(c) It measures the overall standard of living discriminatory access to quality ICT
of a particular country.
(Information and Communications
(d) A higher GDP always denotes a better
Technology) services at economically
distribution of wealth within the
geography of a country. efficient prices to people in rural and
remote areas.

O
64. Consider the following statements with 2. It was created
ed under the Ministry of
respect to the Sea Slugs:

C
Electronics & Information Technology.
1. They are found where there is an
S.
3. The collections of the Universal Service
abundance of prey bases.
Levy (USL) which comprises the USOF,
TE

2. Their presence is an indicator of a strong


coral ecosystem. are credited to the Consolidated Fund of
O

3. They depend on algae for food through India.


photosynthesis.
FN

Which of the statements given above are


Which of the statements given above are
correct?
correct?
D

(a) 1 and 2 only


(a) 1 and 2 only
.P

(b) 2 and 3 only (b) 1 and


and 3 only
W

(c) 1 and 3 only (c) 1, 2 and 3


(d) 1, 2 and 3 (d) 2 and 3 only
W

14 www.visionias.in ©Vision IAS


W
.
67. With reference to GSAT 7 series satellites, 70. Metal injection molding is the
consider the following statements: manufacturing of solid metal parts utilizing

1. They are advanced satellites meant for injection molding technology. Which of the
following products can be made using Metal
meeting the communication needs of
Injection Molding?
defense services.
1. Medical surgical instruments
2. They have been developed by the
2. Computer hinges for laptops
Defence Research and Development
3. Watch cases
Organisation (DRDO). 4. Plugs for cell phones
Which of the statements given above is/are Select the correct answer using the code
correct? given below.
(a) 1 only (a) 1, 2 and 3 only
(b) 2 only (b) 2 and 4 only
(c) 1 and 3 only
(c) Both 1 and 2
(d) 1, 2, 3 and 4
(d) Neither 1 nor 2

71. Which of the following are the major


68. Right to government aid for minority
reforms undertaken as a part of the New
institutions is guaranteed under which article Economic Policy?
of the Constitution of India? 1. Increase in the taxes on individual
(a) Article 29 incomes
(b) Article 30 2. Establishment of private sector banks
(c) Article 21 3. Market-determined exchange rates
4. Deregulation of deposit interest rates
(d) None of the above
Select the correct answer using the code
given below.
69. Cri-MAC, recently seen in the news, is a/an:
(a) 1 and 3 only
(a) mechanism of the Ministry of Home
(b) 1, 2 and 3 only

O
Affairs to share information on heinous (c) 1 and 4 only
crimes and other issues related to inter- (d) 2, 3 and 4 only
C
S.
state coordination.
TE

(b) centre established by the Ministry of 72. "The tendency of the body to repeat its
Social Justice and Empowerment to immune response based on the first variant it
O

track and rehabilitate orphan children. encountered through infection or vaccination


when it comes across a newer or slightly
FN

(c) centre established by the Ministry of


different variant of the same pathogen." The
Science and Technology to promote the
phenomenon in question refers to
D

manufacturing of semi-conductor
(a) Genetic drift
.P

devices. (b) Immune imprinting


W

(d) initiative of the Ministry of External (c) Reverse immunity


Affairs to deport the illegal immigrants. (d) Herd immunity
W

15 www.visionias.in ©Vision IAS


W
.
73. With reference to the cultural history of 76. Which of the following has served as the
India, who among the following belongs to president of Indian National Congress (INC)
the Nirguna School of Bhakti? sessions more than once?
1. Mirabai 1. Madan Mohan Malaviya
2. Nanak 2. Motilal Nehru
3. Kabir 3. Vallabhbhai Patel
4. Chaitanya Select the correct answer using the code
given below.
Select the correct answer using the code
(a) 1 and 2 only
given below.
(b) 2 only
(a) 1, 2 and 3
(c) 1 and 3 only
(b) 2 and 3 only
(d) 1, 2 and 3
(c) 2 only
(d) 1 and 3 only 77. Consider the following statements with
regard to the Mohammedan Literacy
74. Consider the following statements regarding Society:
Clearing Corporations in India: 1. It was founded by Abdul Latif in 1863 in
1. Clearing corporation is established to Calcutta.
undertake the clearing and settlement of 2. The goal of the society was the
trades in securities traded on a education of Muslim youth in English
recognized stock exchange. medium schools that would allow them
2. All clearing corporations in India are to compete with their English and Hindu
regulated by the SEBI. peers.
Which of the statements given above is/are Which of the statements given above is/are
correct? correct?
(a) 1 only (a) 1 only
(b) 2 only
(b) 2 only
(c) Both 1 and 2
(c) Both 1 and 2
(d) Neither 1 nor 2
(d) Neither 1 nor 2

78. Which of the following statements is not


75. With reference to Indian Patent laws,

O
correct regarding the Pangolin species?
consider the following statements: (a) Out of the eight species of pangolin,
1. Derivatives of known substances are
C
only the Indian Pangolin and the
S.
patentable under Indian laws. Chinese Pangolin are naturally found in
2. Patent laws in India does not allow
TE

India.
evergreening of patents. (b) Indian Pangolin is not found in the arid
3. The laws do not grant patent protection North
region, high Himalayas and North-East
O

to pharmaceutical products to ensure India.


FN

their availability to masses at low price. (c) Indian Pangolin is categorised as


Which of the statements given above is/are Pangol
Endangered, where as Chinese Pangolin
D

correct? as Critically Endangered on the IUCN’s


(a) 1 and 2 only red list of threatened species.
.P

(b) 2 only (d) Only Indian Pangolin is legally protected


W

(c) 1 and 3 only under the Wildlife (Protection) Act,


1972.
(d) 1, 2 and 3
W

16 www.visionias.in ©Vision IAS


W
.
79. 'Project Greensand' recently seen in the news 82. With reference to cloud computing, consider
is a: the following statements:
(a) carbon storage project. 1. It is a method of storing and processing
(b) green belt to prevent the eastward data on servers that are remotely located.
expansion of the Thar desert. 2. Cloud computing can be both centralized

(c) sand made from fly ash. and decentralized.

(d) project to increase the green cover in the 3. It is more useful in applications where
large amounts of data need to be
Bundelkhand region.
processed at once.
Which of the statements given above is/are
80. With reference to the “Initiative on Critical
correct?
and Emerging Technologies (iCET)”,
(a) 1 and 2 only
consider the following statements:
(b) 2 and 3 only
1. It is an Indo-U.S. initiative announced at
(c) 3 only
the G20 Summit held in Bali.
(d) 1, 2 and 3
2. It is being run by the National Security
Councils of both the countries. 83. Which one of the following statements best
Which of the statements given above is/are reflects the idea behind the
correct? “Biotransformation technology” often talked
(a) 1 only about in media?
(b) 2 only (a) altering the genetic makeup of Aedes
(c) Both 1 and 2 mosquitos to reduce their population
(d) Neither 1 nor 2 (b) altering the state of plastics and make
them biodegradable
81. Assertion (A): The combined budget of all (c) enhancing the nutritional profile of
the Municipal Corporations in India is much seaweeds

smaller than that of the Central and State (d) developing vaccines for Influenza

O
governments.
84. Consider the following statements regarding
Reason(R): Municipal corporations mostly
rely on municipal bonds than on borrowings
C
the Consumer Food Price Index (CFPI):
S.
1. It is released by National Statistics
from banks, financial institutions, and loans
Office (NSO).
TE

from Centre/State governments.


2. It is released only in two categories rural
In the context of the above two statements,
and urban.
O

which of the following is correct?


3. Its base year is 2016.
FN

(a) Both A and R true but, R is the correct


Which of the statements given above is/are
explanation of A.
correct?
D

(b) Both A and R are true but, R is not the (a) 1 only
.P

correct explanation of A. (b) 1 and 2 only


(c) A is true but R is false. (c) 2 and 3 only
W

(d) A is false but R is true. (d) 1, 2 and 3


W

17 www.visionias.in ©Vision IAS


W
.
85. Consider the following: 88. Consider the following statement with
1. Dragonfly reference to PM-MUDRA yojana (PMMY):
2. Lichens 1. MUDRA is a refinancing institution and
3. Algae Blooms
does not lend directly to entrepreneurs/
4. Frogs
individuals.
Which of the above act as important bio-
2. It aims of provide a collateral free loan
indicators of the ecological health of an
area? of 20 lakh to promote entrepreneurship
(a) 1 and 2 only among the new generation.
(b) 2, 3 and 4 only 3. MUDRA loans are offered in three
(c) 1, 3 and 4 only categories Shishu', 'Kishore' and 'Tarun'
(d) 1, 2, 3 and 4
which signifies the stage of growth or
development and funding needs of the
86. Consider the following statements regarding
borrowers.
the influenza:
1. Influenza viruses consists of single Which of the statement given above is/are
stranded RNA. correct?
2. H3N2 is a Type D influenza virus which (a) 3 only
cause greatest morbidity and mortality. (b) 1 and 3 only
3. Influenza viruses can spread from pigs
(c) 2 and 3 only
to people and from people to pigs.
(d) 2 and 3 only
Which of the statements given above are
correct?
(a) 1 and 3 only 89. The city was called Avantika during the

(b) 1 and 2 only ancient period when it was a primary centre


(c) 2 and 3 only of learning. Great scholars like Varahmihir,
(d) 1, 2 and 3 Kattayayan were associated with the city.

O
King Vikramaditya made this city his
87. Consider the following statements:

C
capital, the great scholar of Sanskrit,
nskrit,
1. Real Exchange Rate determines export
S.
Kaalidas, was in his court. The city also
competitiveness between two countries.
hosts the Kumbh Mela (Simhastha Kumbh)
TE

2. If the currencies of two countries are at


purchasing power parity then the after every 12 years.
O

nominal exchange rate is equal to the Which of the following cities is best
real exchange rate.
FN

described in the paragraph mentioned


Which of the statements given above is/are above?
correct?
D

(a) Varanasi
(a) 1 only
.P

(b) Ujjain
(b) 2 only
(c) Jaipur
W

(c) Both 1 and 2


(d) Neither 1 nor 2 (d) Nasik
W

18 www.visionias.in ©Vision IAS


W
.
90. With reference to Fluoride pollution in 93. With reference to Indian history, which of
groundwater, consider the following the following best describes the ‘Morcha
statements: Chabian’ campaign?
1. Prolonged fluoride consumption in (a) An anti-salt tax march in Bengal region
higher concentrations in drinking water during the Civil Disobedience
can immediately cause diarrhea and
Movement.
impaired nerve function.
(b) An agitation to get the keys back of the
2. Biochar-mediated sand columns can be
Golden Temple treasury back from
used for defluoridation in hand pumps
British appointed officer.
and tube wells.
(c) An armed rebellion in the Chhota
Which of the statements given above is/are
correct? Nagpur region against the rent-seeking
(a) 1 only tendencies of the British rule.
(b) 2 only (d) A campaign launched by the British to
(c) Both 1 and 2 suppress the Ghadarite movement during
(d) Neither 1 nor 2 the first world mar.

91. Which of the following statements best 94. With reference to the death penalty in India,
describes "Kamikaze drones" recently seen consider the following statements:
in the news? 1. Hanging by the neck till death is the
(a) A suicide drone that is believed to have
only execution method mentioned under
been used in the Russia-Ukraine war.
the Code of Criminal Procedure (CrPC),
(b) Surveillance drones used by Pakistan
1973.
along the India-Pakistan border.
2. State of Gujarat has the highest number
(c) Drones used in the United States for the
of convicts on death row in India.
transportation of light-weight materials.
(d) Unmanned Aerial Vehicles that are used Which of the statements given above is/are
in disaster management. correct?
(a) 1 only
92. Consider the following statements regarding (b) 2 only

O
Indian Judicial System: (c) Both 1 and 2
1. The Constitution of India provides for a (d) Neither 1 nor 2
single independent but integrated
C
S.
judicial system, unlike USA's judicial 95. With reference to Indian history, consider
system.
TE

the following texts:


2. While the independent judicial system is 1. Amukthamalyadha
a federal feature of the Constitution, an
2. Harikathasaram
O

integrated judicial system is a unitary


3. Manucharitam
FN

feature.
Which of the above is/are authored by
Which of the statements given above is/are
Krishnadeva Raya?
correct?
D

(a) 1 only (a) 1 only


.P

(b) 2 only (b) 1 and 2 only


(c) 1, 2 and 3
W

(c) Both 1 and 2


(d) Neither 1 nor 2 (d) None
W

19 www.visionias.in ©Vision IAS


W
.
96. The Great Backyard Bird Count (GBBC) 99. Consider the following statements:
report has been released by which of the 1. He was actively involved in the
following organizations? activities of Anushilan Samiti
(a) Bombay Natural History Society 2. He brought out a newspaper called
(b) BirdLife International 'Forward'.
(c) Royal Society for Protection of Birds 3. He is considered to be the political guru
(d) Cornell Lab of Ornithology and National of Subhash Bose.
Audubon Society Which of the following personalities is best
explained by the above-given statements?
97. Consider the following statements: (a) Motilal Nehru
1. Currently, sugarcane production in India (b) Surendranath Banerjee
is more than 400 million tonnes. (c) C. R. Das
2. In India, the production of oilseeds is (d) Bipin Chandra Pal
higher in the Rabi season than Kharif
season. 100. With reference to the universe, which of the
Which of the statements given above is/are following statements is not correct?
correct? (a) The Milk Way is an example of a disc
(a) 1 only shaped spiral galaxy.
(b) 2 only (b) A white dwarf is a very small, hot star,
(c) Both 1 and 2 whose nuclear energy supplies have
(d) Neither 1 nor 2 been used up.
(c) A black dwarf is the first stage of stellar
98. This wildlife sanctuary is located in Odisha. evolution.
It forms a migration link for wild buffaloes (d) Andromeda is the closest big galaxy to
across the two states of Odisha and the Milky Way.
Chhatisgarh. The Chuktia Bhunjia tribes live

O
inside this sanctuary. Recently it was seen in
the news for cultivation of strawberry.
Which of the following wildlife sanctuaries
C
S.
is being referred to in the passage given
TE

above?
(a) Sunabeda Wildlife Santuary
O

(b) Bhitarkanika Wildlife Sanctuary


FN

(c) Gajner Wildlife Sanctuary


(d) Kinnerasani Wildlife Sanctuary
D
.P

Copyright © by Vision IAS


All rights are reserved. No part of this document may be reproduced, stored in a retrieval system or
W

transmitted in any form or by any means, electronic, mechanical, photocopying, recording or otherwise,
without prior permission of Vision IAS.
W

20 www.visionias.in ©Vision IAS


W
.
VISIONIAS
ANSWERS & EXPLANATIONS
GENERAL STUDIES (P) TEST – 3816 (2023)

Q 1.A
• Under Article 105(2) of the Indian Constitution, Members of Parliament enjoy immunity from court
proceedings for their statements in Parliament. However, their speeches are subject to the discipline of the
Rules of Parliament, the "good sense" of its Members, and the control of proceedings by the Speaker.
• Rule 380 of the Rules of Procedure and Conduct of Business in Lok Sabha gives the Speaker the
discretion to expunge any words or expressions used in the debate that are considered defamatory,
indecent, unparliamentary, or undignified. Hence, statement 1 is correct and statement 3 is not correct.
• The expunged portions cease to exist in the records of Parliament and cannot be reported by media
houses, although they may have been heard during the live telecast of the proceedings. Hence, statement
2 is not correct.
• Recently, certain comments from Rahul Gandhi's recent speech in the Lok Sabha have been expunged from
parliamentary records.

Q 2.B
• Recent context: INS Trikand, a Talwar-class frigate of the Indian Navy, participated in Sea Phase-I
of the International Maritime Exercise/Cutlass Express 2023 (IMX/CE-23) held in the Gulf from
March 5 to 9, 2023.
• The exercise is sponsored by US Africa Command and overseen by US Naval Forces Africa (NAVAF), and
almost 50 nations and international maritime organisations are taking part in it.
• During the period, INS Trikand exercised with naval units of Bahrain, Japan, Oman, Saudi Arabia,
UAE, UK and the USA with the common aim of enhancing maritime security, keeping shipping lanes
open, and ensuring the safety of navigation.
• While this is the first time the Indian Navy has participated in IMX, it is also the second time a ship
from the Indian Navy has taken part in a Combined Maritime Force (CMF) exercise. INS Trikand had
previously taken part in Operation Sea Sword 2, which was conducted by the CMF, in November 2022.
• Hence option (b) is the correct answer.

O
Q 3.C

C
• Recent context: A Self-amplifying messenger RNA (mRNA) vaccine, ARCT-154, 154, developed by Arcturus
Therapeutics Holdings (US), showed promising results against Covid19 .
S.
• About mRNA Vaccine:
o Unlike conventional vaccines that inject a weakened form of a virus or bacteria into the body, RNA
TE

vaccines use part of virus’ own genes to stimulate an immune response.


o A conventional mRNA vaccine, such as those from Pfizer and Moderna, uses mRNA that encodes
the spike protein of the coronavirus.
O

✓ mRNA vaccines teach cells how to make copies of the spike protein that triggers an immune
response inside human bodies, when actual infection takes place.
FN

o A self-amplifying
amplifying mRNA vaccine is an improvement over the traditional mRNA as it encodes four
extra proteins in addition to the vaccine antigen, and these enable amplification of original strand of
RNA once inside the cell. Advantage of self-amplifying
amplifying mRNA vaccine over conventional mRNA
D

lowering the cost of vaccines.


vaccine: making storage easy, minimizing dose of RNA, and lowering
.P
W
W

1 www.visionias.in ©Vision IAS


W
.
• DNA and RNA Vaccines:
o In DNA vaccines, a piece of DNA encoding the antigen is first inserted into a bacterial plasmid and
then these DNA plasmids are usually injected into muscles and then enabled to reach cells.
✓ Plasmid is a circular piece of DNA used by a bacterium to store and share genes that may benefit
its survival.
o Whereas, RNA vaccine uses a copy of a natural chemical called messenger RNA (mRNA) to produce
an immune response in humans. Hence statement 2 is correct.
o mRNA teaches cells how to make a protein that triggers an immune response.
o DNA vaccine is more stable than RNA vaccine.
o Unlike RNA vaccines, DNA vaccines have the potential to integrate into the host cell genome. Hence
statement 1 is correct.

Q 4.C
• Kappaphycus alvarezii
o Why in news: The government plans a seaweed park in Tamil Nadu, ignoring the threat that
Kappaphycus alvarezii which is widely grown invasive seaweed, poses to corals in the Gulf of Mannar
Marine National Park running along the state's coastline. Hence option (c) is the correct answer.
o It is seaweed (alga) which is native to the Indo-Pacific region.
o The International Union for Conservation of Nature lists it as one of the world’s 100 most invasive
species.
o It is one of the prime threats that killed the corals near Kurusadai which was deliberately introduced in
Ramanathapuram for commercial cultivation some two decades ago.

Q 5.C
• Protection of Children from Sexual Offences (POCSO) Act, 2012, enacted in consequence to India’s
ratification of the UN Convention on the Rights of the Child in 1992, came into effect on November 14,

O
2012. The aim of this special law is to address offences of sexual exploitation and sexual abuse of children,
which were either not specifically defined or adequately penalised. Hence, statement 1 is correct.


It is a gender-neutral law and not reporting abuse is an offence.
C
Typically, the trauma that child sexual abuse victims endure prevents them from voicing their complaints
S.
immediately. Recognising this, in 2018, the Union Ministry of Law and Justice clarified that there is no
time or age bar for reporting sexual offences under the POCSO Act. Hence, statement 2 is not correct.
TE

• POCSO Act was amended through POCSO (Amendment) Act, 2019 adding Section 14 (Punishment for
using a Child for pornographic purposes) amended to increase the punishment to a minimum of five
O

years and to synchronise with the IT Act, 2000. Hence, statement 3 is correct.
FN

Q 6.D
• Temperature is the most ecologically relevant environmental factor. You are awa aware that the average
temperature on land varies seasonally, decreases progressively from the equator towards the poles and from
D

plains to the mountain tops. It ranges from subzero levels in polar areas and high altitudes to > 500 ℃ in
tropical deserts in summer.
.P

• There are, however, unique habitats such as thermal springs and deep-
deep-sea
deep -sea
sea hydrothermal vents where average
temperatures exceed 1000 C. It is general knowledge that mango trees do not and cannot grow in temperate
W

countries like Canada and Germany, snow w leopards are not found in Kerala forests and tuna fish are rarely
caught beyond tropical latitudes in the ocean. You can readily appreciate the significance of temperature to
W

2 www.visionias.in ©Vision IAS


W
.
living organisms when you realise that it affects the kinetics of enzymes and through it the basal metabolism,
activity and other physiological functions of the organism.
• A few organisms can tolerate and thrive in a wide range of temperatures (they are called eurythermal),
Hence, statement 1 is not correct.
• But, a vast majority of them are restricted to a narrow range of temperatures (such organisms are
called stenothermal). Hence, statement 2 is not correct.
• The levels of thermal tolerance of different species determine to a large extent their geographical
distribution. Hence, statement 3 is correct.

Q 7.A
• Trafficking of children and women for commercial sexual exploitation is an organized crime violating basic
human rights. Lack of a protective environment, low status of women, and poverty are a few reasons for
trafficking.
• Ujjawala scheme launched by the Central Government in 2007, for the prevention of illicit trafficking
and also for the rescue, reintegration, and rehabilitation of the victims who were trafficked for
commercial sexual exploitation. Hence statement 1 is correct.
• It is a centrally sponsored scheme, launched by the Ministry of Women and Child Development. Hence
statement 2 is not correct.

Q 8.C
• Cruise Missile:
o An unmanned self-propelled (till the time of impact) guided vehicle that sustains flight through
aerodynamic lift for most of its flight path.
o They fly within the earth’s atmosphere and use jet engine technology. Hence statement 1 is correct.
o Example: BrahMos, Harpoon (USA), Exocet (France)
o Classification:
✓ Subsonic (around 0.8 Mach speed), Supersonic (around 2-3 Mach speed), Hypersonic (More than
5 Mach speed)
• Ballistic Missile:
o Has a ballistic trajectory over most of its flight path, regardless of whether it is a weapon-delivery
vehicle.
o Powered by rockets initially but then they follow an unpowered, free-falling trajectory toward their
targets. Hence statement 2 is correct.
o Example: Prithvi I, Prithvi II, Agni I, Agni II and Dhanush ballistic missiles .
o Classification: Based on launch mode, range, propulsion system.

O
C
S.
TE
O

Q 9.C
FN

• Central Board of Film Certification (CBFC) is a statutory body under the Ministry of Information
and Broadcasting, regulating the public exhibition of films under the provisions of the
Cinematograph Act 1952. Hence, statement 1 is correct.
D

• Films can be publicly exhibited in India only afterr they have been certified by the Central Board of Film
.P

Certification.
• The Board consists of non-official
official members and a Chairman (all of whom are appointed by the Central
W

Government) and functions with headquarters in Mumbai. It has nine Regional offices, one each at Mumbai,
Kolkata, Chennai, Bangalore, Thiruvananthapuram, Hyderabad, New Delhi, Cuttack, and Guwahati.
W

3 www.visionias.in ©Vision IAS


W
.
• The Regional Offices are assisted in the examination of films by Advisory Panels. The members of the
panels are nominated by the Central Government by drawing people from different walks of life for
a period of 2 years. Hence, statement 2 is correct.
• The Certification process is in accordance with The Cinematograph Act, 1952, The Cinematograph
(certification) Rules, 1983, and the guidelines issued by the Central government u/s 5 (B)
o U- Unrestricted Public Exhibition
o U/A-Unrestricted Public Exhibition - but with a word of caution that discretion is required for children
below 12 years
o A- Restricted to adults
o S-Restricted to any special class of persons.

Q 10.B
• The summoning of Parliament is specified in Article 85 of the Constitution. The power to convene a session
of Parliament rests with the government. The decision is taken by the Cabinet Committee on Parliamentary
Affairs. This decision is formalized by the President, in whose name MPs are summoned to meet for a
session. During the first two decades of Parliament, Lok Sabha met for an average of a little more than 120
days a year. This has come down to approximately 70 days in the last decade.
• India does not have a fixed parliamentary calendar. By convention, Parliament meets for three
sessions in a year the longest, being the Budget Session, the Monsoon Session, and the Winter Session.
It is only stated in Article 85 of the constitution that there should not be a gap of more than six months
between two sessions of Parliament. Hence, statement 2 is not correct.
• The 14th and 15th Lok Sabha saw 60% and 71% of bills referred to committees. This number has
dipped sharply to just 27% in the 16th Lok Sabha and just 11% in the 17th Lok Sabha (2019-
2021). Hence, statement 1 is correct.
• The National Commission to Review the Working of the Constitution has recommended the minimum
number of sittings for Lok Sabha and Rajya Sabha be fixed at 120 and 100 respectively.

Q 11.A
• TiHAN (Technology Innovation Hub on Autonomous Navigation) is India’s first Autonomous
Navigation facility for both ground and aerial vehicle testing. It recently was inaugurated at IIT
Hyderabad. The testbed will provide industries, research and development labs, and academia with a
platform for research in autonomous navigation. It will be the destination for next-generation safe,
sustainable and smart mobility solutions. The country’s first testbed for autonomous navigation would
provide a platform for industries, research and development labs and academia to drive collaborative
research in autonomous navigation. Hence, statement 1 is correct.
• TiHAN is a multi-departmental initiative, including researchers from Electrical Engineering, Computer
Science and Engineering, and Mechanical and Aerospace Engineering at IIT-Hyderabad. Hence,
statement 2 is not correct.

Q 12.D

O
• Iron Catastrophe and Planetary differentiation:
o When Earth was formed about 4.5 billion years ago, it was a uniform ball of hot rock. Radioactive decay

C
and leftover heat from planetary formation (the collision, accretion, and compression of space rocks)
caused the ball to get even hotter. Eventually, after about 500 million years, our young planet’s
S.
temperature heated to the melting point of iron — about 1,538° Celsius. This pivotal moment in Earth’s
history is called the iron catastrophe. Hence option (d) is the correct answer.
TE

o The iron catastrophe allowed greater, more rapid movement of Earth’s molten, rocky material.
Relatively
tively buoyant material, such as silicates, water, and even air, stayed close to the planet’s exterior.
These materials became the early mantle and crust. Droplets of iron, nickel, and other heavy metals
O

gravitated to the centre of Earth, becoming the early y core. This important process is called planetary
differentiation.
FN

Q 13.A
• Devaluation is the deliberate downward adjustment of the value of a country's money relative to
D

another currency, group of currencies, or currency standard. Currency devaluation iinvolves taking
.P

measures to strategically lower the purchasing power of a nation's own currency.


• It may seem counter-intuitive,
intuitive, but a strong currency is not necessarily in a nation's best interests. A weak
W

domestic currency makes a nation's exports more competitive


petitive in global markets, and simultaneously makes
imports more expensive.
W

4 www.visionias.in ©Vision IAS


W
.
• Higher export volumes spur economic growth, while pricey imports also have a similar effect because
consumers opt for local alternatives to imported products. This improvement in the terms of trade generally
translates into a lower current account deficit (or a greater current account surplus), higher employment,
and faster GDP growth. The stimulative monetary policies that usually result in a weak currency also have
a positive impact on the nation's capital and housing markets, which in turn boosts domestic consumption
through the wealth effect. Hence statement 1 is correct.
• It is worth noting that a strategic currency devaluation does not always work, and moreover may lead to a
'currency war' between nations. Competitive devaluation is a specific scenario in which one nation matches
an abrupt national currency devaluation with another currency devaluation. In other words, one nation is
matched by a currency devaluation of another.
• Currency devaluation may lower productivity, since imports of capital equipment and machinery
may become too expensive. Devaluation also significantly reduces the overseas purchasing power of a
nation’s citizens.
• Devaluation also increases the debt burden of foreign-denominated loans when priced in the home
currency. This is a big problem for a developing country like India or Argentina which hold lots of dollars-
and euro-denominated debt. These foreign debts become more difficult to service, reducing confidence
among the people in their domestic currency. Hence, statement 2 is not correct.
• There are many downsides to currency devaluation, like rising inflation, and more costly foreign debt
servicing. It further reduces the confidence of foreign investors in the country’s currency as well.

Q 14.D
• Hail stones are a frozen form of precipitation that occurs when thunderstorm updrafts lift rain above
the freezing level in the atmosphere. When hail stones become too heavy to be lifted by the updraft, they
fall to the ground.
• While most thunderstorms form hail, hail does not always make it all the way to the ground as hail. If the
hail stone is small enough, the stone melts in the warmest portions of the atmosphere which are near the
ground. For hail to become large enough to reach the surface, the stone must grow large enough in the
thunderstorm before falling through the warm lower atmosphere.
• Hail forms when a thunderstorm updraft lifts a water droplet above the freezing level in the
atmosphere. The frozen water droplet then accretes super-cooled water or water vapor, which freezes once
it comes in contact with the frozen droplet. This process causes a hailstone to grow.
o Supercooled water is something unique–it’s water that is below its normal freezing point of 0°C (32°F)
and yet remains a liquid.
• Hail is often confused with other types of freezing precipitation such as sleet. Sleet is found mainly
during the cold season and does not occur in thunderstorms. Hail, in comparison, is only found in
thunderstorms where updrafts in the thunderstorm forces raindrops further up in the atmosphere to freeze.
• In many cases, hailstones have a ringed appearance. The rings represent the different environments the
hailstone experiences while moving through the updraft. When the hailstone is in an environment where
mainly water vapor is present, a white or opaque layer forms. This occurs because small air pockets are
trapped between the vapor particles as they freeze. When the hailstone is in an environment of mainly

O
super-cooled water, a clear layer forms as the super-cooled water freezes instantaneously to the hailstone.
• Hailstones can also grow by sticking to each other in a process called wet growth. Larger hailstones will

C
ascend through the updraft at a slower speed than smaller hailstones. If the outer coating of these hailstones
is not completely frozen, they can collide with each other and stick. If this process happens over and over
S.
again, a hailstone can grow very quickly. When these aggregated hailstones hitt the ground, they often have
a bumpy or spiky appearance (right), as the smaller hailstones that make up the larger hailstone maintain
TE

their individual shapes.


• Hail sizes can differ greatly from one storm to another depending on the strength of the storm’s
storm’ s updraft.
Stronger updrafts can create larger hailstones, which in turn causes more damage.
O

• Favourable Conditions For Hail Formation:


o Strong, upward motion of air (strong updraft) within the parent thunderstorm
FN

o High liquid water content


o Great vertical extent of the cumulonimbus cloud
o Good portion of the cloud layer is below freezing (oC or below)
D

o High surface temperatures (hail growth is greatly inhibited during cold surface temperatures)
.P

o Hence option (d) is the correct answer.


W
W

5 www.visionias.in ©Vision IAS


W
.
Q 15.A
• Wheat is the main cereal crop in India. Its cultivation in India has traditionally been dominated by the
northern region of India. Major wheat-growing states in India are Uttar Pradesh, Punjab, Haryana, Madhya
Pradesh, Rajasthan, Bihar and Gujarat.
o India is today the second-largest wheat producer in the whole world.
o India is also world's second-biggest consumer of wheat. Hence statement 1 is correct.
• Indian wheat is largely a soft/medium hard, medium protein, white bread wheat, somewhat similar to U.S.
hard white wheat. Wheat grown in central and western India is typically hard, with high protein and high
gluten content.
• Wheat crop has wide adaptability. It can be grown not only in the tropical and sub-tropical zones, but also
in the temperate zone and the cold tracts of the far north, beyond even the 60 degree north altitude . Wheat
can tolerate severe cold and snow and resume growth with the setting in of warm weather in spring. It can
be cultivated from sea level to as high as 3300 meters.
• The best wheat are produced in areas favoured with cool, moist weather during the major portion of
the growing period followed by dry, warm weather to enable the grain to ripen properly.
o The optimum temperature range for ideal germination of wheat seed is 20-25 degree Celsius though the
seeds can germinate in the temperature range 3.5 to 35 c. Rains just after sowing hamper germination
and encourage seedling blight. Areas with a warm and damp climate are not suited for wheat growing.
o During the heading and flowering stages, excessively high or low temperatures and drought are harmful
to wheat.
o Cloudy weather, with high humidity and low temperatures is conducive for rust attack. Wheat plant
requires about 14-15 c optimum average temperature at the time of ripening . The temperature
conditions at the time of grain filling and development are very crucial for yield. Temperatures above
25 c during this period tend to depress grain weight. When temperatures are high, too much energy is
lost through the process of transpiration by the plants and the reduced residual energy results in poorer
grain formation and lower yields. Wheat is mainly a rabi (winter) season crop in India.
• Wheat is grown in a variety of soils of India. Soils with a clay loam or loam texture, good structure and
moderate water holding capacity are ideal for wheat cultivation. Care should be taken to avoid very porous
and excessively drained oils. Soil should be neutral in its reaction.
o Heavy soil with good drainage are suitable for wheat cultivation under dry conditions. These soils
absorb and retain rain water well. Hence statement 3 is correct.
o Heavy soils with poor structure and poor drainage are not suitable as wheat is sensitive to water logging.
o Wheat can be successfully grown on lighter soils provided their water and nutrient holding capacity are
improved.
• World trade in wheat is greater than for all other crops combined. The demand for India's wheat in the
world shows a rising trend. The country has exported 7,239,366.80 MT of wheat to the world for the worth
of Rs. 15,840.31 crores/ 2,121.72 USD Millions during the year 2021-22. Hence statement 2 is correct.
o Major Export Destinations (2021-22) : Bangladesh, Sri Lanka, United Arab EMTs, Yemen Republic,
Philippines and Indonesia.

O
Q 16.D
• In addition to making of the Constitution
on and enacting ordinary laws, the Constituent Assembly also

C
performed the following functions:
o It ratified India’s membership of the Commonwealth in May 1949.
S.
o It adopted the national flag on July 22, 1947. Hence statement 1 is not correct
o It adopted the national anthem on January 24, 1950. Hence statement 2 is correct.
TE

o It adopted the national song on January 24, 1950. Hence statement 3 is correct
o It elected Dr. Rajendra Prasad as the first President of India on January 24, 1950. Hence
statement 4 is correct
O

Q 17.A
FN

• India has three cadres of CHWs.


o The first created is the Auxiliary Nurse-Midwife (ANM), who is based at a sub-center
sub and visits
villages in addition to providing care at the subcenter.
D

o The second is the Anganwadi Worker (AWW), who works solely in her village and focuses on
.P

provision of food supplements to young children, adolescent girls, and lactating women.
o The third is the Accredited Social Health Activist (ASHA), who also works solely in her village.
W

ASHA workers focus on promotion of Maternal and Child Health (MCH), including immunizations
and institutional-based
based deliveries, for which they receive a performance
performance-related fee.
W

6 www.visionias.in ©Vision IAS


W
.
• Anganwadi Services is a component of the Umbrella Integrated Child Development Services scheme
of the Ministry of Women and Child Development along with PM Matru Vandana Yojana; POSHAN
Abhiyan; Scheme for Adolescent girls; Child Protection Scheme and National Creche Scheme. Hence
statement 2 is correct.
o The beneficiaries under the Scheme are children in the age group of 0-6 years, pregnant women and
lactating mothers.
o It offers a package of six services, viz. supplementary nutrition; pre-school non-formal education;
nutrition & health education; immunization; health check-ups and referral services.
✓ Three of the six services namely Immunization, Health Check-ups and Referral Services delivered
through Public Health Infrastructure under the Ministry of Health & Family Welfare.
o Anganwadi Workers (AWW) hold regular meetings with ASHAs and provide on-job training by
discussing the activities undertaken. Hence statement 1 is correct.
✓ Anganwadi Worker (AWW) guides ASHAs in performing the following activities:
▪ Anganwadi worker will be depot holder for drug kits and will be issuing them to ASHA.
▪ AWWs and ANMs will act as resource persons for the training of ASHA.
▪ Anganwadi worker will be depot holder for drug kits and will be issuing it to ASHA.
▪ ASHA will support the AWW in mobilizing pregnant and lactating women and infants for
nutrition supplements.
• An auxiliary nurse midwife or nurse hybrid commonly known as ANM is a village-level female health
worker in India who is known as the first contact person between the community and the health
services.
• ANMs are positioned as a key health worker within the National Rural/Urban Health Mission human
resources framework.
• ASHA Worker:
o ASHAs are community-based functionaries under National Rural Health Mission (NRHM).
o ASHA is the first port of call for any health-related demands of deprived sections of the population,
especially women and children, who find it difficult to access health services.
o ASHA must be primarily a woman resident of the village- ‘Married/ Widow/Divorced’ and preferably
in the age group of 25 to 45 yrs.
o She should be a literate woman with formal education up to Eighth Class. This may be relaxed only if
no suitable person with this qualification is available.
o ASHA also acts as a depot holder for essential provisions being made available to every habitation like
Oral Rehydration Therapy (ORS), Iron Folic Acid Tablet (IFA), chloroquine, Disposable Delivery Kits
(DDK), Oral Pills & Condoms, etc.
✓ A Drug Kit will be provided to each ASHA. She also provides primary medical care for minor
ailments such as diarrhoea, fevers, and first aid for minor injuries.
✓ She will be a provider of Directly Observed Treatment Short-course (DOTS) under the Revised
National Tuberculosis Control Programme.

O
C
S.
TE
O
FN
D
.P
W

• Hence, statement 3 is not correct.


W

7 www.visionias.in ©Vision IAS


W
.
Q 18.D
• Tsunamis are characterized as shallow-water waves. Shallow-water waves are different from wind-
generated waves, the waves many of us have observed at the beach. Wind-generated waves usually have
period (time between two successional waves) of five to twenty seconds and a wavelength (distance between
two successional waves) of about 100 to 200 meters. A tsunami can have a period in the range of ten minutes
to two hours and a wavelength in excess of 300 miles (500 km). It is because of their long wavelengths that
tsunamis behave as shallow-water waves.

• The rate at which a wave loses its energy is inversely related to its wavelength. Since a tsunami has a
very large wavelength, it will lose little energy as it propagates. Hence in very deep water, a tsunami will
travel at high speeds and travel great transoceanic distances with limited energy loss. For example, when
the ocean is 20,000 ft. (6100 m) deep, unnoticed tsunamis travel about 550 miles per hour (890 km/hr), the
speed of a jet airplane. And they can move from one side of the Pacific Ocean to the other side in less than
one day. Hence statement 2 is correct.
• As a tsunami leaves the deep water of the open sea and propagates into the more shallow waters near
the coast, it undergoes a transformation. Since the speed of the tsunami is related to the water depth,
as the depth of the water decreases, the speed of the tsunami diminishes. The change of total energy
of the tsunami remains constant. Therefore, the speed of the tsunami decreases as it enters shallower
water, and the height of the wave grows. Because of this “shoaling” effect, a tsunami that was imperceptible
in deep water may grow to be several feet or more in height. Hence statement 1 is correct.
• Tsunami Ready: It is a community performance-based programme initiated by the Intergovernmental
Oceanographic Commission (IOC) of UNESCO to promote tsunami preparedness through active
collaboration of public, community leaders, and national and local emergency management agencies. Hence
statement 3 is correct.

Q 19.A
• Recent context: The Taiwan visit of Nancy Pelosi, Speaker of the US House of Representatives, in
August, 2022 triggered a strong reaction from the Chinese government, which conducted "live fire"
exercises in the Taiwan Strait for an unprecedented five days in a row.
• The Island Chain Strategy was formulated in 1951 by John Foster Dulles, a foreign affairs expert who
enumerated the need to contain the erstwhile USSR and China with a string of naval bases in the
western Pacific region to restrict sea access to these two countries.
o This strategy did not gain much traction during the Cold War, but after the USSR broke up into 15

O
countries on December 26, 1991, the Island Chain Strategy gained momentum to contain China,
whose growing economic prowess and its desire for hegemony in the Indo-Pacific Pacific region were no

C
secret.
• An important dimension of this strategygy is the strategic location and importance of Taiwan, which China
S.
wants to conquer and take back to its folds.
• The strategy encompasses three island chains - thehe First Island Chain, the Second Island Chain and the
TE

Third Island Chain, all three of which are in the Pacific Ocean.
o The First Island Chain comprises the Kuril Islands, the Japanese Archipelago, the Ryukyu
Islands, Taiwan, the northwest Philippines and ends at Borneo. This chain is also the first line of
O

defence and serves as the maritime boundaries ies between the East China Sea and the Philippine Sea and
FN

the South China Sea and the Sulu Sea.


o The Second Island Chain consists of the Bonin Islands, Volcano Islands, Mariana Islands, western
Caroline Islands and Western New Guinea.
D

o The Third Island Chain refers to the Aleutianleutian Islands running through the centre of the Pacific
Ocean through the Hawaiian Islands, American Samoa, and Fiji, culminating at New Zealand.
.P

• Of late, it has been proposed to create a Fourth and a Fifth Island Chain in the Indian Ocea Ocean.
o The Fourth Island Chain proposes to have the Lakshadweep Islands Islands, Maldives, Diego Garcia,
W

with Gwadar and Hambantota in Pakistan and Sri Lanka respectively.


W

8 www.visionias.in ©Vision IAS


W
.
o The Fifth Island Chain is planned to originate from Camp Lemonnier in the Gulf of Aden, travel
around the Horn of Africa, along the East African coastline, and through the Mozambique
Channel towards South Africa with the aim of encircling the Chinese naval bases in Doraleh and
Djibouti.
• Thus, the five Island Chains of the Island Chain Strategy are expected to checkmate China and contain its
dreams for control over the Indo-Pacific region.

• Hence option (a) is the correct answer.

Q 20.C
• Babur, the founder of Mughal rule in India (1526), ruled for only four years. He was not able to contribute
anything to the growth of painting. His successor, Humayun, was mostly engaged in containing his rivals
until he was forced out of India by Sher Shah in 1540. It was, however, during his refuge at the court of
Shah Tahmasp of Persia that Humayun acquired a love for the art of painting.
• Humayun was so influenced by the art practiced there that he commissioned Mir Sayyid Ali and
Khwaja Abdus Samad, two Persian masters, to illustrate manuscripts for him. These two painters
joined Humayun’s entourage on his triumphant return to India. Hence statement 1 is correct.
• Akbar gave liberal patronage to the growth of fine arts during his rule. The first major project undertaken

O
during Akbar’s regime was that of illustrating the Hamza Nama. It began in 1562 for which several
artists were employed at the court. The place where the painters worked was known as Tasvir Khana. Hence
statement 2 is not correct.
C

S.
Painting under Akbar’s reign distinguishes itself as a tradition from Persian painting as well as from
Indian styles, particularly by the presence of historical subject matter. It became recognizable within
a span of fifteen years since the setting up of royal atelier under Akbar. By about 1590, it had acquired a
TE

distinctive form, marked by:


o naturalism and rhythm,
O

o clothing objects of daily use assuming Indian forms,


o picture space having subsidiary scenes set in the background,
FN

o extraordinary vigor of action and violent movement, and


o luxuriant depiction of foliage and brilliant blossoms.
• It should be emphasized here that the identity
tity of the Mughal paintings under Akbar was as much made
D

of an original style as a fusion of the Persian and Indian traditions. The depiction of action and
movement is not to be found in either the pre-Mughal
Mughal art of India or the art of Persia. Hence statement 3
.P

is correct.
• The two most commonly used themes were: daily events of the court, and portraits of leading personalities.
W
W

9 www.visionias.in ©Vision IAS


W
.
Q 21.A
• Sixth Five Year Plan
o The Plan focussed on Increase in national income, modernization of technology, ensuring continuous
decrease in poverty and unemployment through schemes for transferring skills (TRYSEM) and
providing slack season employment (NREP), controlling population explosion etc.
o Broadly, the sixth Plan could be taken as a success as most of the target were realised even though
during the last year (1984-85) many parts of the country faced severe famine conditions and agricultural
output was less than the record output of previous year.
o GOI launched Drought Prone Area Development Programme (DPAP) during 1973-74 (Fourth
Five Year Plan) to address special problems of drought-prone areas. Hence option (a) is the
correct answer.
• Fifth Five Year Plan
o The final Draft of fifth plan was prepared and launched by D.P. Dhar in the backdrop of economic crisis
arising out of run-away inflation fuelled by hike in oil price.
o It proposed to achieve two main objectives: 'removal of poverty' (Garibi Hatao) and 'attainment
of self reliance'.
o Promotion of high rate of growth, better distribution of income and significant growth in the domestic
rate of savings were seen as key instruments.
o The target growth rate was 4.4% and the actual growth rate turned out to be 4.8%.
• Rolling Plan (1978-80)
o This was a period of instability. The Janata Party government rejected the fifth five-year Plan and
introduced a new Sixth Five-Year Plan. This, in turn, was rejected by the Indian National Congress in
1980 upon Indira Gandhi's re-election.
o A rolling plan is one in which the effectiveness of the plan is evaluated annually and a new plan is
created the following year based on this evaluation. As a result, throughout this plan, both the allocation
and the targets are updated.
• Eighth Five Year Plan
o The eighth plan was postponed by two years because of political uncertainty at the Centre. Worsening
Balance of Payment position, rising debt burden, widening budget deficits, recession in industry and
inflation were the key issues during the launch of the plan.
o The plan undertook drastic policy measures to combat the bad economic situation and to undertake an
annual average growth of 5.6% through introduction of fiscal & economic reforms including
liberalization under the Prime Ministership of Shri P V Narasimha Rao.
o Some of the main economic outcomes during the eighth plan period were rapid economic growth
(highest annual growth rate so far – 6.8 %), high growth of agriculture and allied sector, and
manufacturing sector, growth in exports and imports, improvement in trade and current account deficit.
o A high growth rate was achieved even though the share of public sector in total investment had declined
considerably to about 34 %.
o Target Growth was 5.6 % but Actual Growth is 6.8%.
• Eleventh Five-Year Plan

O
o India had emerged as one of the fastest growing economy by the end of the Tenth Plan. The savings
and investment rates had increased,, industrial sector had responded well to face competition in the

C
global economy and foreign investors were keen to invest in India.
o But the growth was not perceived as sufficiently inclusive for many groups, specially SCs,, STs &
S.
minorities
orities as borne out by data on several dimensions like poverty, malnutrition, mortality, current daily
employment etc.
TE

o Eleventh Plan was aimed “Towards Faster & More Inclusive Growth “.
o Target growth was 9 % actual growth is 8%.
O

Q 22.D
• The Hoysala Empire was spread over the Southern Deccan Plateau region of present-day
present Karnataka
FN

between the 11th and 14th centuries. It developed a distinctive style of Hindu temple architectur
architecture known
as Hoysala architecture.. This architectural style is exemplified by numerous
numerous temples, both large and small,
built during the 13th century, including the Chennakesava Temple at Belur,
Belur, the Hoysaleswara Temple
D

at Halebidu, and the Kesava Temple at Somanathapura mostly concentrated in southern Karnataka.
.P

Hence statement 1 is correct.


• The Hoysala temples are quite unique in that their different parts are merged together to form a unified,
W

organic whole, unlike the temples of the Tamil region where each part of a temple stands independently.
Although they may appear superficially different,
rent, Hoysala temples share a similar structural design. They
W

10 www.visionias.in ©Vision IAS


W
.
are renowned for their intricate and detailed sculptures, which decorate all parts of the temple, and
are carved out of soft soapstone (chloritic schist), a material that is ideal for fine and detailed work. Local
craftsmen were primarily responsible for executing these carvings, which display architectural features that
set them apart from other temple architectures in South India. Hence statement 2 is not correct.
• The Hoysala temples are known for their unique and complex structure, featuring multiple shrines
arranged around a central pillared hall in a star-shaped pattern. This stellate-plan design sets them
apart from other medieval temples in South India. Also, these sport a pyramidical Vimana as its Shikhar.
These distinctive architectural features and decorative elements make the Hoysala temples easily
recognizable and distinguishable from other temple architectures of the time. Hence statements 3 and 4
are correct.

Q 23.B
• Ways and Means Advances (WMAs) are introduced in 1997. They are temporary advances given by
RBI (under the RBI Act 1934) to Centre and States to tide over any mismatch in receipts and payments.
o Such advances are repayable within three months from the date of making that advance. Interest is
charged at existing repo rate (rate at which RBI ends short-term money to banks).
• WMA is not part of the Fiscal Responsibility and Budget Management Act (FRBM), 2003 because
they get paid within the year itself. Hence statement 1 is correct.
• Two types of WMA –special and normal
o Special WMA or Special Drawing Facility is provided against the collateral of the government
securities held by the state. After the state has exhausted the limit of SDF, it gets normal WMA.
o The interest rate for SDF is one percentage point less than the repo rate. Hence statement 2 is not
correct.
• There is a state-wise limit for the funds that can be availed via WMA. Decided by the government and
RBI mutually, these limits depend on factors such as total expenditure, revenue deficit and fiscal position
of the State. Hence statement 3 is correct.

Q 24.C
• India’s international trade crossed the $1 trillion mark for the first time in the year ending December
2022. However, data from the Ministry of Commerce and Industry show that this has come with a
record-high trade deficit — over $85 billion — with China. Hence, statement 1 is not correct.
• An analysis of the official data also highlights the rising concentration of the neighboring country in India’s
overall imports over the past 10 years. According to the data for January-December 2022, with exports
worth $453 billion and imports to the tune of $723 billion, India’s overall international trade reached $1.17
trillion, 21 percent above the previous year’s $969 billion. India’s trade deficit — excess of imports over
exports — stands at $270 billion, about 51 percent higher than $178 billion in 2021.
• An analysis of India’s foreign trade statistics shows that while international trade has risen by 15 percent
on average in the past 10 years, both India’s exports and imports have become slightly more
concentrated, or limited to fewer countries.
• Another interesting finding from these statistics is the slight rise in the concentration of India’s import

O
partners. For instance, in 2012, the top 10 import partners used to account for 53 percent of India’s overall
imports. In 2022, the share of the top 10 import partners has increased to 58 percent. This is largely due to

C
China’s rising share in India’s overall imports. In 2012, Chinese
ese goods accounted for 10.6 percent of India’s
overall imports. Data available on the Ministry of Commerce and Industry’s web portal Tradestat shows
S.
wi th a
that the US is India’s biggest trade partner with a total trade value of $131 billion. China is second, with
total trade value of $118 billion. These two nations alone accounted for more than a fifth of India’s
TE

international trade.
• The graph given below shows the top export destination for Indian India. It can be seen that Bangladesh
is one of the top export destinations of India. Hence, statement 2 is not correct.
O
FN
D
.P
W
W

11 www.visionias.in ©Vision IAS


W
.
Q 25.D
• The Cartaz system referred to a naval trade license or pass issued by the Portuguese in Indian ocean
during the sixteenth century.
• Under the cartaz system, the Portuguese exacted money from the traders as price for protection against what
they termed as piracy. But much of this was caused by Portuguese freebooters themselves and so the whole
system was a blatant protection racket.
• The Cartaz was typically a permit valid for one year and obtained after paying a fee for its issue. The fee
varied depending on the ships destination, and was obviously higher for overseas trade.
• Overseas Cartazes were issued only in Goa (after the ship reached there with a coastal Cartaz) and
Cartazes for coastal trade could be obtained from any Portuguese fort or factory or possession. The
Portuguese armada controlling these seas had absolute control and every king, lord or emperor followed the
rules, except of course the moors of Calicut and Ponnani who never agreed with the policy (Cochin, Quilon
and Cannanore agreed). The Portuguese used the system effectively on all Indian rulers, until the other
western powers like the Dutch and English made their appearances in the 17th century.
• Hence option (d) is the correct answer.

O
Q 26.B
• The Ministry of Housing & Urban Affairs has initiated Affordable Rental Housing Complexes

C
(ARHCs). It is a sub-scheme under the Pradhan Mantri AWAS Yojana- Urban (PMAY-U). ). This will
provide ease of living to urban migrants/poor in the Industrial Sector as well as in the non-formal
formal urban
S.
economy to get access to dignified affordable rental housing close to their workplace. Hence statement 2
is correct and 4 is not correct.
TE

• The beneficiaries for ARHCs will be urban migrants/poor from EWS/LIG IG categories including labor,
urban poor (street vendors, rickshaw pullers, other service providers, etc.), industrial workers, migrants
working with market/ trade associations, educational/ health institutions, hospitality sector, long term
O

tourists/visitors, students or any other persons of such category. Hence statement 3 is correct.
FN

• The Scheme was hence announced under the Atmanirbhar Bharat Package which compliments the
"Housing for all" motto of the government. Hence statement 1 is correct.
D

Q 27.B
• There are several west-flowing
flowing rivers in India that originate in the Western Ghats and flow westwards
.P

flowing rivers in India are:


towards the Arabian Sea. Some of the major west-flowing
o Narmada River: It is one of the longest rivers in India, flowing through the sstates of Madhya Pradesh,
W

Gujarat, and Maharashtra. The river originates in Amarkantak and flows westward into the Arabian
Sea.
W

12 www.visionias.in ©Vision IAS


W
.
o Tapi River: It is also known as the Tapti River, and it flows through the states of Maharashtra, Gujarat,
and Madhya Pradesh. The river originates in the Satpura Range and flows westward into the
Arabian Sea.
o The Luni River originates in the Aravalli Range in Rajasthan and flows through the Thar Desert
before emptying into the Rann of Kutch in Gujarat.
o Sabarmati River: It originates in the Aravalli Range and flows through the states of Rajasthan and
Gujarat before emptying into the Arabian Sea.
o Mahi River: It flows through the states of Madhya Pradesh, Rajasthan, and Gujarat before emptying
into the Arabian Sea.
o Zuari River: It is a river in Goa that originates in the Western Ghats and flows westward into the
Arabian Sea.
o The Periyar River originates in the Western Ghats in Kerala and flows westward into the Arabian
Sea. It is the longest river in the state of Kerala and an important source of water for irrigation and
hydroelectric power generation.
o Kaveri (also known as Cauvery) river is not a west-flowing river in India. It is actually a river that
flows in a predominantly eastward direction, originating in the Western Ghats in Karnataka and flowing
through Tamil Nadu and Puducherry before emptying into the Bay of Bengal on the eastern coast of
India.
• Hence option (b) is the correct answer.

Q 28.B
• In order to refer to a particular location on the earth, it is divided with the help of imaginary horizontal
and vertical lines known as latitudes and longitudes respectively.
• The latitude denotes a geographical coordinate of a place located on the surface of the earth. It is
the angular distance of any point (north or south of the equator) measured in degrees with respect to the
Centre of the Earth. Its value is zero at the equator and 90 degrees at the poles. Hence statement 1 is
not correct.
• As the earth is slightly flattened at the poles, the linear distance of a degree of latitude at the pole is slightly
longer than that at the equator. For example at the equator (0°) it is 68.704 miles, at 45° it is 69.054 miles
and at the poles, it is 69.407 miles. The average distance between two latitudes is thus taken as 69 miles
(111km). Hence, statement 2 is correct.
• Some significant latitudes include-
o Equator (0°).
o North pole (90°N) and the south pole (90° S).
o Tropic of Cancer (23½° N).
o Tropic of Capricorn (23½° S).
o Arctic circle at 66½° N.
o Antarctic circle at 66½° S.
• Longitude, on the other hand, is an angular distance measured in degrees along the equator east or
west of the Prime Meridian.

O
• In 1884, the zero meridian was chosen as the one passing through the Royal Astronomical Observatory
at Greenwich, near London. This is the Prime Meridian (0°) from which all other meridians radiate

C
eastwards and westwards up to 180°.
• The meridians of longitude converge at the poles.
S.
• Apart from describing the location, they have one more essential function, they determine local time in
relation to G.M.T. or Greenwich Mean Time.
TE
O
FN
D
.P
W
W

13 www.visionias.in ©Vision IAS


W
.
Q 29.D
• Fifteenth Finance Commission chairman NK Singh has called for a thorough review of the Seventh
Schedule of the Constitution in the wake of the current challenges of climate change and the pandemic.
Article 246 confers legislative powers on the Parliament and the State Legislatures on the subjects
enumerated in the Seventh Schedule.
• In its 1988 report, Sarkaria Commission dedicated an entire chapter to legislative relations, recording the
grievances raised by various State Governments and political parties. Its major recommendations were:
o First, the residuary powers be transferred from the Union List to the Concurrent List, except for
the residuary power to impose taxes which should be retained in the Union List.
o Second, the States should be consulted by the Centre before the latter exercises its power over
Concurrent List entries.
o Third, the Centre should limit the field it occupies with respect to Concurrent List entries to only as
much as is necessary for ensuring uniformity in basic issues of national policy, with the details being
left for state action.
✓ M.M. Punchhi commission, in 2010 recommended that the Union should only transfer those
subjects into the Concurrent List, which is central to achieving demonstrable national interest.
✓ The 1971 report of the Rajamannar Committee, formally known as the Centre-State Relations
Inquiry Committee, put it thus: “The Committee is of the opinion that it is desirable to constitute a
High Power Commission, consisting of eminent lawyers and jurists and elderly statesmen with
administrative experience to examine the entries of Lists I and III in the Seventh Schedule to the
Constitution and suggest a redistribution of the entries. Hence option (d) is the correct answer.

Q 30.C
• Agra Fort, also called Red Fort, large 16th-century fortress of red sandstone located on the Yamuna
River in the historic city of Agra, west-central Uttar Pradesh, north-central India. It was established by
the Mughal emperor Akbar and, in its capacity as both a military base and a royal residence, served
as the seat of government when the Mughal capital was in Agra. Hence, statement 1 is correct.
• The walls of the roughly crescent-shaped structure have a circumference of about 1.5 miles (2.5 km), rise
70 feet (21 meters) high, and are surrounded by a moat. There are two access points in the walls: the Amar
Singh Gate facing south (now the only means in or out of the fort complex) and the Delhi Gate facing west,
the original entrance, which is richly decorated with intricate marble inlays. Many structures within the
walls were added later by subsequent Mughal emperors, notably Shah Jahān and Jahāngīr. The complex of
buildings—reminiscent of Persian- and Timurid-style architectural features—forms a city within a city.
• The structure, a contemporary of Humāyūn’s Tomb in Delhi, reflects the architectural grandeur of the
Mughal reign in India.
• The Agra fort complex was designated a UNESCO World Heritage site in 1983. Hence, statement 2
is correct.

Q 31.B
• The Department of Commerce under Union Ministry of Commerce and Industry, Government of

O
India, is implementing the Trade Infrastructure for Export Scheme (TIES) w.e.f. FY 2017-18 to assist
Central and State Government agencies in creating appropriate infrastructure for the growth of exports.xports.

C
Hence statement 3 is correct.
• Under the scheme, financial assistance in the form of grants-in-aid aid is provided to Central/State
S.
Government owned agencies (or their Joint Ventures with major stake-holding holding by them) for setting
up or up-grading export infrastructure in States/UTs. Hence statement 1 is not correct.
TE

• The States can avail of the scheme through their implementing agencies for infrastructure projects with
significant export linkages like Border Haats, Land customs stations, quality testing andd certification labs,
cold chains, trade promotion centres, export warehousing and packaging, SEZs and ports/airports cargo
O

terminuses.
• Negative List of Projects that will not be considered under this Scheme:
FN

o Projects which are covered under sector specificic schemes like textiles, electronics, IT.
o General infrastructure projects like highways, power etc. Hence statement 2 is correct.
o Projects where an overwhelming export linkage cannot be established.
D
.P

Q 32.A
• The Edicts of Ashoka are 33 inscriptions engraved d on pillars, large stones, and cave walls by Ashoka the
W

Great (r. 268-232


232 BCE), the third king of the Mauryan Empire (322 (322-185 BCE) of India. One set, the so-
called Major Rock Edicts, are consistent in their message that the people should adhere to the con
concept of
W

14 www.visionias.in ©Vision IAS


W
.
Dhamma, defined as “right behavior”, “good conduct” and “decency toward others”. The edicts were
inscribed throughout Ashoka's realm which included the areas of modern-day Afghanistan, Bangladesh,
India, Nepal, and Pakistan and most were written in Brahmi Script (though one, in Afghanistan, is also
given in Kharoshti script). Hence, statement 2 is not correct.
• A series of results that he published between 1836 and 1838 Prinsep was able to decipher the inscriptions
on rock edicts found around India. The edicts in Brahmi script mentioned a King Devanampriya Piyadasi
which Prinsep initially assumed was a Sri Lankan king. He was then able to associate this title with Ashoka
on the basis of Pali script from Sri Lanka communicated to him by George Turnour. These scripts were
found on the pillars at Delhi and Allahabad and on rock inscriptions from both sides of India, and also the
Kharosthi script in the coins and inscriptions of the northwest. Hence, statement 1 is correct.

Q 33.D
• Nitrate radical is an oxide of nitrogen that consists of three oxygen atoms bound to a nitrogen atom. It arises
from the oxidation of nitrogen dioxide (NO2) by ozone (O3). Hence, statement 1 is correct.
• The Nitrate Radical is a strong oxidant, reacting with a wide variety of volatile organic compounds,
including alkenes, aromatics, and oxygenates as well as with reduced sulfur compounds, which will then
generate ozone and secondary organic aerosol. So this deteriorates air quality. Hence, statement 2 is
correct.
• Nitrate Radicals occur principally in the nighttime atmosphere due to its rapid photolysis in sunlight and
its reaction with Nitric Oxide (NO). Hence, statement 3 is correct.
• Recently, a new study published in Nature Geosciences has found that parts of India and China are
hotspots for the night-time production of nitrate radicals that can increase the amount of deadly ozone
and PM2.5 particulate matter in the atmosphere.

Q 34.D
• Polar stratospheric clouds (PSCs) are clouds that form in the winter polar stratosphere at altitudes
ranging from 12 km – 22 km (49,000 to 82,000 feet) above the surface. They are most visible during
twilight when the Sun is about 1-6 degrees below the horizon, as well as in winter and at higher latitudes.
• They belong to the category of nacreous clouds, sometimes called mother-of-pearl clouds.
o These are rare clouds mostly visible within two hours after sunset or before dawn.
o They form at temperatures of around -85°C, which is colder than average lower stratosphere
temperatures, and are comprised of ice particles ~10µm across. Hence statement 2 is correct.
o The clouds must be composed of similar-sized crystals in order to produce the characteristic bright
iridescent colors by diffraction and interference.
• They are mostly spotted during winter at high latitudes like Scandinavia, Iceland, Alaska, and Northern
Canada. Hence statement 3 is correct.
• Ozone depletion- These high-altitude clouds form only at very low temperatures and help destroy ozone
in two ways:
o They provide a surface that converts benign forms of chlorine into reactive, ozone-destroying
forms,

O
o They remove nitrogen compounds that moderate the destructive impact of chlorine. Hence
statement 1 is correct.

C
S.
TE
O
FN
D
.P
W
W

15 www.visionias.in ©Vision IAS


W
.
• In recent years, the atmosphere above the Arctic has been colder than usual, and polar stratospheric
clouds have lasted into the spring. As a result, ozone levels have been decreasing.
• There are two distinct types of Polar Stratospheric Clouds (which can be measured using LIDAR). :
o Type I clouds- composed of nitric acid and water (and sometimes also sulfuric acid),
o Type II clouds contain only water ice.

Q 35.C
Comparison of FPTP and PR system of election
• India adopted the First Past the Post (FPTP) voting method for elections in Lok Sabha and State
Assemblies since Independence.
• First past the post or FPTP, also known as Simple Majority Voting, Winner-takes-all voting, or Plurality
voting is the most basic form of the voting system in which the country is divided into small geographical
called constituencies or districts.
• While every constituency elects one representative, the voter votes for the candidate, not for the party.
• In this voting method, the candidate with the highest number of votes in a constituency is declared the
winner.
o There is no minimum threshold limit to cross. Even a margin of one vote results in victory.
o Thus, the FPTP system allows the candidate to win despite securing less than half the votes in a contest.
o Also, smaller parties representing specific groups have a lower chance of being elected in FPTP.
Hence, statement 1 is not correct.
o A candidate who wins the election may not get the majority (50%+1) votes and thus a party may
get more seats than votes in the legislature. Hence, statement 2 is correct.
o Examples: U.K., India
• Proportional representation (PR) refers to election systems in which electorate divisions are
proportionally represented in the elected body. If X% of the electorate supports a political party, that party
will win around X% of the seats in the legislature.
• The foundation of such systems is that all votes, not just a plurality or a simple majority, count toward the
conclusion.
• Every party gets seats in the legislature in proportion to the percentage of votes that it gets and the voter
votes for the party.
• More than one representative may be elected from one constituency and a candidate who wins the
election gets the majority of votes. Hence, statement 3 is correct.
• Large geographical areas are demarcated as constituencies. The entire country may be a single constituency.
• Examples: Israel, Netherlands

Q 36.D
• The Karkota Dynasty governed the Kashmir valley and certain northern regions of the Indian subcontinent
during the 7th and 8th centuries. Their reign was characterised by political growth, economic affluence,
and the rise of Kashmir as a hub of learning and culture.
• In 625 CE, Durlabhvardhana established the Karkota Dynasty, which is also known as Prajnaditya and

O
during his rule, Hsuan Tsang visited Kashmir. According to Rajatarangini, there were no instances of

C
military operations during his tenure. Hence statement 1 is correct.
• The Karkota rulers constructed numerous Vishnu temples. They also permitted the growth of Buddhism
S.
during their reign, as evidenced by the presence of Stupas, Chaityas, and Viharas in the remains of their
capital city. Lalitaditya constructed the Martand Sun Temple in the Anantnag district, which is
TE

the earliest known Sun temple in India and was one of the most expansive temple complexes of that
era. Hence statement 2 is correct.
• With the ascent of Avanti Varman to the throne of Kashmir circa 855 CE, the Utpala dynasty came into
O

being, superseding the Karkota dynasty's rule. Hence statement 3 is correct.


• Hence option (d) is the correct answer.
FN

Q 37.D
• Singphos Rebellion
D

o Bom Singpho was a prominent member of the Beesa Gaum. He was one one of the ring leaders who joined
.P

with some of the Ahom nobles to overthrow British rule in Assam.


o On 25 March 1830, the rebel group of 400 men under the leadership of Peali Borgohain advanced
W

towards Rangpur (near Sibsagar, Assam) and attacked the continge contingent of 30 sepoys under a
Jamadar who were guarding the armory depot.
W

16 www.visionias.in ©Vision IAS


W
.
o Later on, the ring leaders of the rebel group including Bom Singpho and his compatriots were captured,
except Peali Brogohain who managed to escape into the Naga Hills.
o Later, they were found guilty of treason and sentenced to death. However, the death sentence for Bom
Singpho was commuted to 14 years of rigorous imprisonment for his involvement in a rebellion against
the British Government.
o Also, the Singhpos lived around the Lohit river region and the Patkai Bum. Thus, the rebellion took
place in North-East India. Hence pair 1 is correctly matched.
• Kherwar Movement of Jharkhand
o Kherwar Movement started apparently in 1868 and rose to fame under the leadership of Bhagrit
Manjhi. It is also known as the Sapha Har movement. This movement popularized the concept of One
God as well as aimed at social reform. He demanded that a Santal raj should be introduced. He wanted
the Santhal Tribes to stop their sacrifices to the "evil bongas".
o He was arrested in that same year. Bhagrit Manjhi was arrested in 1868 as an instigator of disturbances.
o He was also sentenced to two years imprisonment. Several Hindu leaders participated. Thus, the
movement took place in Jharkhand (Eastern India). Hence pair 2 is correctly matched.
• Manyam Rebellion
o The Rampa Rebellion of 1922, also known as the Manyam Rebellion, was a tribal uprising led by
Alluri Sitarama Raju in Godavari Agency of Madras Presidency, British India.
o The Rampa administrative area, situated in the hills of the present Godavari districts of Andhra Pradesh,
comprised around 700 square miles and comprised mostly of the Tribal population. They had
traditionally supported their food requirements, through the use, in particular, of the Podu system,
whereby each year some areas of jungle forest were burned to clear land for cultivation.
o The British wanted to take control of the forest land for building railways and ships.
o Alluri Sitarama Raju, a sanyasi, a person of justice and strong willpower, raised his voice against the
unlawful British Policy.
o The revolt started in August 1922 and ended in May 1924 after the capture and killing of Raju.
• Hence pair 3 is correctly matched.

Q 38.D
• The Indian plate (Indo-Australian Plate) includes Peninsular India and the Australian continental portions.
• The subduction zone along the Himalayas forms the northern plate boundary in the form of continent—
continent convergence.
• In the east, it extends through Rakinyoma Mountains of Myanmar towards the island arc along the Java
Trench. The eastern margin is a convergent boundary lying to the east of Australia in the form of an oceanic
ridge in SW Pacific.
• The Western margin follows Kirthar Mountain of Pakistan. It further extends along the Makrana coast and
joins the spreading site from the Red Sea rift south-eastward along the Chagos Archipelago.
o Red Sea rift is formed due to the divergence of Somali Plate and Arabian Plate. Hence option (d)
is the correct answer.
o Chagos Archipelago formed due to hotspot volcanism.

O
• The boundary between India and the Antarctic plate is also marked by oceanic ridge (divergent boundary)
running in roughly W-E direction and merging into the spreading site, a little south of New Zealand.

C
S.
TE
O
FN
D
.P
W
W

17 www.visionias.in ©Vision IAS


W
.
Q 39.D
• Recent context: India and Egypt agreed in January, 2023, to elevate their bilateral ties to "Strategic
Partnership" covering political, security, defence, energy and economic aspects.
• "Strategic relationship" is one of the most frequently used phrases in foreign policy discussions today, but
perhaps one of the least understood. Scholars have traced its appearance in international relations to the end
of the Cold War. Countries that were until then arranged in blocs allied to one of the two superpowers
suddenly found themselves on their own and began to cast about for new bilateral alliances, usually with
states more powerful than themselves.
• In the past two decades, India has signed strategic partnership agreements with more than 30 countries,
including the US, Russia, France, the UK, Germany, and Japan.
• Strategic partnerships are commonly associated with defence or security-related issues, but a survey of
formal strategic partnerships around the world reveals they can also be quite a hold-all, covering a wide

O
range of bilateral relations, from defence to education,
tion, health and agriculture, and quite commonly,
economic relations, including trade, investment and banking.
• The relationship is characterised by mutual trust, understanding, and support and is aimed at
C
S.
achieving mutual benefits and addressing common challenges. Strategic partnerships go beyond formal
agreements on specific issues or economic partnerships focused on trade and investment, and they are not
TE

necessarily limited to forming military alliances to counter a common enemy.


• About Egypt:
o It links north-east Africa with the Middle East.
O

o It occupies a crucial geo-strategic location — around 12% of global trade passes through the Suez
Canal.
FN

o In 1961, India and Egypt along with Yugoslavia, Indonesia and Ghana established the Non Non-Aligned
Movement (NAM).
o It has been a non-permanent
permanent member of United Nations Security Council.
D

o The first joint special forces exercise between the Indian Army and the Egyptian Army, "Exercise
Cyclone-I"
I" has been underway since 14 January 2023 in Jaisalmer, Rajasthan.
.P
W
W

18 www.visionias.in ©Vision IAS


W
.
• Hence option (d) is the correct answer.

Q 40.A
• Created in 1995, the World Trade Organization (WTO) is an international institution that oversees
the rules for global trade among nations. It superseded the 1947 General Agreement on Tariffs and
Trade (GATT) created in the wake of World War II.
• The WTO is based on agreements signed by a majority of the world’s trading nations. The main function
of the organization is to help producers of goods and services, as well as exporters and importers, protect
and manage their businesses.
• As of 2021, the WTO has 164 member countries, with Liberia and Afghanistan the most recent
members, having joined in July 2016, and 25 “observer” countries and governments.
• Russian Federation became a member of WTO on 22 August 2012.
• Ukraine became a member of WTO on 16 May 2008
• Iran is not a member of WTO. But has an observer status.

O
C
S.
TE
O
FN
D
.P
W

• Hence option (a) is the correct answer.


W

19 www.visionias.in ©Vision IAS


W
.
Q 41.C
• Menander I was a Greco-Bactrian and later Indo-Greek King (reigned c.165/155[4] –130 BC) who
administered a large territory in the Northwestern regions of the Indian Subcontinent from his capital at
Sagala (Sakala). Menander is noted for having become a patron and converted to Greco-Buddhism and he
is widely regarded as the greatest of the Indo-Greek kings.
• Menander is believed to have been initially a king of Bactria. After re-conquering Punjab he established
an empire that stretched from the Kabul River valley in the west to the Ravi River in the east, and from the
Swat River valley in the north to Arachosia (the Helmand Province). Ancient Indian writers indicate that he
launched expeditions southward into Rajasthan and as far east down the Ganges River Valley as Pataliputra
(Patna), and the Greek geographer Strabo wrote that he "conquered more tribes than Alexander the Great."
• Large numbers of Menander’s coins have been unearthed, attesting to both the flourishing commerce and
longevity of his realm. Menander was also a patron of Buddhism, and his conversations with the Buddhist
sage Nagasena are recorded in the important Buddhist work, the Milinda Panha ("The Questions of
King Milinda"; panha meaning "question" in Pali). After his death in 130 BC, he was succeeded by his
wife Agathocleia, perhaps the daughter of Agathocles, who ruled as regent for his son Strato I. Buddhist
tradition relates that he handed over his kingdom to his son and retired from the world, but Plutarch says
that he died in camp while on a military campaign and that his remains were divided equally between the
cities to be enshrined in monuments, probably stupas, across his realm.
• Hence, option (c) is the correct answer.
• Rudradaman I was a Śaka ruler from the Western Kshatrapas dynasty. He was the grandson of king
Caṣṭana. Rudradāman I was instrumental in the decline of the Sātavāhana Empire. Rudradāman I took up
the title of Maha-kshtrapa, after he became the king and then strengthened his kingdom.
• Gondophares (Gondophernes) I was the founder of the Indo-Parthian Kingdom and its most prominent
king, ruling from 19 to 46. He probably belonged to a line of local princes who had governed the Parthian
province of Drangiana since its disruption by the Indo-Scythians in c. 129 BC.
• Demetrius I Anicetus, also called Damaytra was a Greco-Bactrian and later Indo-Greek king, who ruled
areas from Bactria to ancient northwestern India.

Q 42.C
• Coffee Berry Borer
o Why in News: The Coffee Board has issued an advisory to coffee farmers to tackle the attack of berry
borer in Robusta coffee plants.
o Coffee berry borers are small species of beetle that live, feed, and breed inside coffee berries.
o The damage to the berry severely impacts the quality of the coffee beans produced.
o Infestation: A pinhole at the tip of berries indicates the presence of the pest.
o Female beetles bore into the berries through the navel region and feed on the inside content.
o Infested berries may fall due to injury or secondary infection.
o Severe infestation may result in heavy crop loss.
o Control Measure: Brocca traps are an effective measure to control the berry borer population in
coffee plantations. Hence, option (c) is the correct answer.

O
Q 43.B

C
• Local Winds are the winds that usually affect small areas and are confined to the lower levels of the
troposphere. These are caused by the air moving between high and low-pressure systems in confined
S.
spaces.
• Harmattan is observed in Western African nations. It occurs in-between the end of November and the
TE

middle of March month.


• It is usually a cool dry wind blowing from the northeast or east in the western Sahara. Due to
its arid origins, it usually carries a large amount of dust, which it can transport hundreds of miles out
O

over the Atlantic Ocean. Depending on the local circumstances the temperature during harmattan can
range from cold in most places to hot in certain regions.
FN

• Due to its strong dryness,, as compared to the humid tropical air, it carries an ability to kill pests and
germs. Hence, it is popularly known as the “doctor wind”.
• It is known for its desert-like conditions because of the following characteristics:
D

o lowering the humidity,


.P

o dissipation of cloud cover,


o prevention of rainfall and
W

o sometimes leads to the occurrence of desert storms or sandstorms.


sandstorms
W

20 www.visionias.in ©Vision IAS


W
.
• Some of the drawbacks associated with the Harmattan include an increase in the fire risk because of the
dryness which can also cause severe damage to the crops. Also, the interaction of the Harmattan with the
monsoon winds can cause tornadoes.

Q 44.D
• Estuaries are transitional zones between the sea and rivers where freshwater streams or rivers merge
with the ocean. The temperature and water currents keep on changing considerably on a seasonal, daily
and even hourly basis.
• In the estuaries, the water composition keeps on changing due to tidal action and water runoff from the land.
Due to the accumulation of organic materials and agricultural chemicals from adjacent land into the water,
the level of nutrients is also high.
• Estuaries are very important to the lives of many animal species. They are often called the “nurseries of
the sea” because numerous animal species rely on estuaries for nesting and breeding.
• Flora: The flora includes seaweeds, marsh grasses, mangrove trees. The primary producers are
phytoplankton, which is consumed directly by some fish. However, most of these phytoplanktons form the
primary food for small crustaceans and the zooplankton, which in turn are food for fish.
• Fauna: Estuaries constitute a variety of fauna including worms, oysters, crabs, and waterfowls. The
estuarine animals show a number of adaptations to the unstable conditions of the environment. They have
a special ability to maintain salt and water balance in the presence of a changing environment. They

O
also have special adaptations to tidal and ocean action.
• Hence, all the statements are correct.

C
S.
Q 45.D
• Priority Sector means those sectors which are considered as important for the development of the basic
TE

needs of the country and are to be given priority over other sectors.
• Priority sector lending (PSL) should constitute 40 percent of the Adjusted Net Bank Credit.
• Priority Sector includes the following categories:
O

o Agriculture
o Renewable Energy
FN

o Micro, Small and Medium Enterprises


o Export Credit
o Education Loans
D

o Housing
o Social Infrastructure
.P

o Advances to weaker sections


✓ Loans to minorities,, women, scheduled caste and scheduled tribes, small and marginal farmers,
W

self-help groups, cottage industries etc.


W

21 www.visionias.in ©Vision IAS


W
.
✓ Overdrafts upto ₹ 5,000/- under Pradhan Mantri Jan-Dhan Yojana (PMJDY) accounts,
provided the borrower’s household annual income does not exceed ₹ 100,000/- for rural areas and
₹ 1,60,000/- for non-rural areas,
• If there is any shortfall in the achievement of target, banks may be required to invest in:
o Funds with NABARD or National Housing Bank (NHB) or Small Industries Development Bank of
India (SIDBI) such as Rural Infrastructure Development Fund (RIDF)
o Micro Units Development Refinance Agency Bank (MUDRA Ltd)
o Priority sector lending certificates (PSLC)
• Hence option (d) is the correct answer.
Q 46.C
• The Union Territory of Jammu and Kashmir was carved out from the erstwhile State of Jammu and Kashmir
through the Jammu and Kashmir Reorganisation Act, 2019. The delimitation of Assembly Constituencies
in the erstwhile State of Jammu and Kashmir was governed by the Constitution of the State of Jammu and
Kashmir and the Jammu and Kashmir Representation of People Act 1957. The Assembly seats in the
erstwhile state of Jammu and Kashmir were last delimited in 1995 based on the 1981 Census.
• The Delimitation Commission is headed by Justice Ranjana Prakash Desai, (a retired Judge of the Supreme
Court of India), and Sh. Sushil Chandra, (Chief Election Commissioner) and Sh. K. K. Sharma, (State
Election Commissioner, Union Territory of Jammu and Kashmir), as an Ex-Officio member of the
Delimitation Commission recently finalized the Delimitation Order for the Union Territory of Jammu &
Kashmir.
• As per the final Delimitation Order, the following will come into effect from the date to be notified by the
Central Government:
o Out of the 90 Assembly Constituencies in the region, 43 will be part of the Jammu region and 47 for
the Kashmir region keeping in view the provisions of Section 9(1)(a) of the Delimitation Act, 2002 and
Section 60(2)(b) of Jammu & Kashmir Reorganization Act, 2019.
o After consultation with Associate Members, representatives of political parties, citizens, and civil
society groups, 9ACs have been reserved for STs, out of which,6 are in the Jammu region and 3
ACs in the Valley. Accordingly, the Delimitation Commission has reserved nine ACs for STs for
the first time and 07 for SCs. It is worthwhile to mention that the Constitution of erstwhile Jammu
and Kashmir State did not provide for the reservation of seats for the Scheduled Tribes in the
Legislative Assembly. Hence, statement 3 is not correct.
o There are five Parliamentary Constituencies in the region. The Delimitation Commission has seen the
Jammu & Kashmir region as one single Union Territory.
o It may be recalled that the Delimitation Commission was constituted by the Govt. of India, in the
exercise of powers conferred by Section 3 of the Delimitation Act, 2002 (33 of 2002), for the
purpose of delimitation of Assembly and Parliamentary Constituencies in the Union Territory of
Jammu & Kashmir. Hence, statement 1 is correct.
o The Delimitation Commission was entrusted with the work of delimiting the Assembly and
Parliamentary Constituencies in the UT of Jammu and Kashmir on the basis of the 2011 Census
and in accordance with the provisions of Part-V of the Jammu and Kashmir Reorganisation Act,

O
2019 and the provisions of Delimitation Act, 2002. Hence, statement 2 is correct.
• The Delimitation Commission also made the following recommendations to the Central Government.

C
o Provision of at least two members (one of them must be a female) from the community of Kashmiri
Migrants in the Legislative Assembly and such members may be given power at par with the power of
S.
nominated members, of the Legislative Assembly of Union Territory of Puducherry.
o The Central Government may consider giving the Displaced Persons from Pakistan-occupied occupied Jammu
TE

and Kashmir some representation in the Jammu and Kashmir Legislative Assembly, by way of the
nomination of representatives of the Displaced
isplaced Persons from Pakistan Occupied Jammu and Kashmir.
Q 47.C
O

• The heart is a muscular organ which is as big as our fist. Because both oxygen and carbon dioxide have to
oxygen-rich
be transported by the blood, the heart has different chambers to prevent the oxygen-
oxygen -rich
rich blood from mixing
FN

with the blood containing carbon dioxide. The carbon dioxide-richrich blood has to reach the lungs for the
carbon dioxide to be removed, and the oxygenated blood from the lungs has to be brought back to
the heart. This oxygen-rich blood is then pumped to the rest of the body.
D

• The separation of the right side and the left side of the heart is useful to keep oxygenated and
.P

deoxygenated blood from mixing.. Such separation allows a highly efficient supply of oxygen to the body.
This is useful in animals that have high energy needs, such as birds and mammals, which constantly use
W

energy to maintain their body temperature.


• Hence, both statements are correct.
W

22 www.visionias.in ©Vision IAS


W
W
W
W
.P
D
FN
O
TE
S.
C
O .
W
W
W
.P
D
FN
O
TE
S.
C
O .
W
W
W
.P
D
FN
O
TE
S.
C
O .
.
✓ If anyone does so they must report it to the nearest police station or the authorised officer within
48-hours of obtaining such possession.
✓ The burden of proof - Under Section 57 of the Act, if a person is found in possession, custody, or
control of any wildlife, the burden of proof for establishing that it is not illegal is on the person.

Q 55.D
• The Bahmani Kingdom was a medieval Muslim state that emerged in the Deccan region of India in
the 14th century. The kingdom was founded by Hasan Gangu Bahmani, who broke away from the Delhi
Sultanate to establish an independent state in the south.
• Malik-ul-Tujjar was a prominent position given to Mahmud Gawan in the Bahmani Kingdom. The title
literally means "King or chief of Merchants". Hence statement 1 is correct.
• The Bahmani Kingdom was divided into various provinces which were called taraf. The governors of
these tarafs were known as tarafdars. (Taraf means province and Atraf is plural of Taraf). Hence statement
2 is correct.
• Dabul was of great importance in the 14th, 15th, and 16th centuries. It used to be the principal port of the
South Konkan region, carrying on trade with ports in the Mediterranean, the Red Sea, and the Persian
Gulf. During the 13th to 15th centuries, this port was ruled by the Bahamani dynasty and was known
as Mustafabad then Hamjabad and then it was Dabhol. Hence statement 3 is correct.
• Khalisa land was that piece of land that was used to run the expenses of the king and the royal
household. Nobles used to get their salary either in cash or in form of a grant of land or jagir.

Q 56.A
• The power to create new districts or alter or abolish existing districts rests with the State governments. This
can either be done through an executive order or by passing a law in the State Assembly. States prefer the
executive route by simply issuing a notification in the official gazette. Hence, statements 1 and 2 are
correct.
• States argue that smaller districts lead to better administration and governance. For example, in 2016, the
Assam government issued a notification to upgrade the Majuli sub-division to the Majuli district for
“administrative expediency”.
• The Centre has no role to play in the alteration of districts or the creation of new ones. States are free to
decide. The Home Ministry comes into the picture when a State wants to change the name of a district or a
railway station. The State government’s request is sent to other departments and agencies such as the
Ministry of Earth Sciences, Intelligence Bureau, Department of Posts, Geographical Survey of India
Sciences and the Railway Ministry seeking clearance. A no-objection certificate may be issued after
examining their replies. Hence, statement 3 is not correct.
• According to the 2011 Census, there were 593 districts in the country. As of 2022, there is a total of 766
districts in India together in 28 states and Union territories.

Q 57.C
• Section 6A of the Citizenship Act, 1955 deals with “special provisions as to the citizenship of persons

O
covered by the Assam Accord”.
• Assam is the only state in India to have an exclusive cut-off date for citizenship.. For the rest of the

C
country, the cut-off date if July 19, 1949.
• It says that all those who came to Assam on or after January 1, 1966, but before March 25, 1971, from
S.
the specified territory (it includes all territories of Bangladesh at thee time of commencement of the
Citizenship (Amendment) Act, 1985), and since then are residents of Assam, must register themselves under
TE

Section 18 for citizenship.


• Hence, option (c) is the correct answer.
• Recently, a plea was filed before the constitutionall bench, while questioning the constitutional validity of
O

Section 6 A, wants 1951 to be established as the cut-off off date for inclusion in the National Register of
Citizens instead of 1971.
FN

Q 58.A
• Recently, the Reserve Bank of India has announced that all eligible ligible current account transactions
D

including trade transactions with Sri Lanka may be settled in any permitted currency outside the
.P

Asian Clearing Union (ACU) mechanism until further notice.


• Asian Clearing Union (ACU)
ACU) is a payment arrangement whereby the participants settle payments for intra
intra-
W

regional transactions among the participating central banks on a net multilateral basis.
• The objectives of the ACU are:
W

26 www.visionias.in ©Vision IAS


W
.
o To provide a facility to settle payments, on a multilateral basis, for current international transactions
among the territories of participants
o To promote the use of participants' currencies in current transactions between their respective territories
and thereby effect economies in the use of the participants' exchange reserves
o To promote monetary co-operation among the participants and closer relations among the banking
systems in their territories and thereby contribute to the expansion of trade and economic activity among
the countries of the ESCAP region
o To provide for currency SWAP arrangement among the participants so as to make Asian Monetary
Units (AMUs) available to them temporarily.
• Headquartered in Tehran, Iran, the ACU was established on December 9, 1974 at the initiative of the
United Nations Economic and Social Commission for Asia and Pacific (ESCAP), for promoting
regional cooperation. Hence statement 1 is correct.
• Asian Monetary Unit (AMU) is the common unit of account of ACU and is denominated as ‘ACU
dollar’, ‘ACU euro’ and ‘ACU yen’, which is equivalent in value to one US dollar, one euro and one
Japanese yen respectively.
o All instruments of payments under ACU have to be denominated in AMUs. Thus, majority of
transactions are not carried out in domestic currencies.
o Hence statement 2 is not correct.
• Settlement of such instruments is made by Category-I banks through the ACU dollar accounts, ACU euro
accounts and ACU yen accounts, which should be distinct from the other US dollar, euro and Japanese yen
accounts respectively maintained for non-ACU transactions.
• Central banks and monetary authorities of Bangladesh, Bhutan, India, Iran, Maldives, Myanmar,
Nepal, Pakistan and Sri Lanka are currently members of the ACU. Hence statement 3 is not correct.

Q 59.B
• Anekantavada, (Sanskrit: “non-one-sidedness” or “many-sidedness”) in Jainism, the ontological
assumption that any entity is at once enduring but also undergoing change that is both constant and
inevitable. Hence option (b) is the correct answer.
o The doctrine of anekantavada states that all entities have three aspects: substance (dravya), quality
(guna), and mode (paryaya). Dravya serves as a substratum for multiple gunas, each of which is itself
constantly undergoing transformation or modification. Thus, any entity has both an abiding continuous
nature and qualities that are in a state of constant flux.
• Mādhyamaka, otherwise known as Śūnyavāda and Niḥsvabhāvavāda, refers to a tradition of Buddhist
philosophy and practice founded by the Indian Buddhist monk and philosopher Nāgārjuna.
o The foundational text of the Mādhyamaka tradition is Nāgārjuna's Mūlamadhyamakakārikā.
• The Sarvāstivāda was one of the early Buddhist schools established around the reign of Ashoka (3rd
century BCE). It was particularly known as an Abhidharma tradition, with a unique set of seven
Abhidharma works.
• Anatta (Anattavada), (Pali: “non-self” or “substanceless”) Sanskrit anatman, in Buddhism, the
doctrine that there is in humans no permanent, underlying substance that can be called the soul.

O
Q 60.D

C
• The organization of the Chief Labour Commissioner (Central) also known n as Central Industrial Relations
Machinery is an apex organization in the country responsible for maintaining harmonious industrial
S.
relations mainly in the sphere of central Government.
• The Organisation of Chief Labour Commissioner (Central) (CLC(C) is entrusted trusted with the following
TE

functions
o Prevention and settlement of industrial disputes through conciliation/mediation. Hence statement
1 is correct.
O

o enforcement of awards and settlements


o implementation of labour laws in industries and establishments in respect
ect of which Central Government
FN

is the appropriate government


o verification of membership of Trade Unions affiliated to the Central Organisations of workers for
giving them representation in national and international conferences and committees. Hence statement
D

2 is correct.
.P

o fixation and revision of dearness allowance component of minimum wages under the Minimum
Wages Act, 1948 in the scheduled employments. Hence statement 3 is correct.
W

o Assists Labour Ministry in preparation of different reports required to be submitted to ILO.


o Attending to Parliamentary Committees and other important delegations as per advice of the Ministry.
W

27 www.visionias.in ©Vision IAS


W
.
Q 61.A
• Flexible fuel vehicles (FFVs) have an internal combustion engine. The internal combustion engine
(spark-ignited) is one of the important components of the FFVs. The fuel is injected into either the intake
manifold or the combustion chamber, where it is combined with air, and the air/fuel mixture is ignited by
the spark from a spark plug.
• Flexible fuel vehicles (FFVs) are capable of operating on gasoline or methanol/ethanol. They can
operate on any blend of gasoline and ethanol up to 83%. For instance, flex-fuel (E85) is a gasoline-ethanol
blend containing 51% to 83% ethanol, depending on geography and season. Hence, statement 1 is correct.
• Key Components of a Flex Fuel Car include a battery, Electronic control module (ECM), Fuel filler,
etc. The battery provides electricity to start the engine and power vehicle electronics/accessories.
• Flex-fuel vehicles (have single tanks) are distinguished from bi-fuel vehicles, where two fuels are stored
in separate tanks and the engine runs on one fuel at a time, for example, compressed natural gas (CNG),
liquefied petroleum gas (LPG), or hydrogen. Hence statement 2 is not correct.

Q 62.B
• Recently, the Supreme Court had an all-woman bench which was only the third time in its history. Only
three times (2013, 2018 2022) in the history of the Supreme Court all women benches were formed.
• This third all-women bench heard transfer petitions involving matrimonial disputes and bail matters. Hence,
statement 1 is not correct.
• The first all-women bench was set up in 2013 when a bench of Justices Gyan Sudha Misra and Ranjana
Prakash Desai was constituted followed by a bench of Justices R. Banumathi and Indira Banerjee in 2018.
There are three women judges in the top court at present including Justice Kohli, B.V. Nagarathna, and
Justice Trivedi. Justice Nagarathna is also set to become the first woman Chief Justice in 2027.
• Recently, in the first Supreme Court, 9 judges including three women took oath in one go.
• Until now, right from 1950, there have been only eleven women judges. In 1980, Justice M Fathima Beevi
became the first woman judge to be appointed to the apex court. Hence, statement 2 is correct.
• 76 out of the 650 occupied High Court judges are currently women. This amounts to 11.7% of all the current
judges of the 25 High Courts in India.

Q 63.A
• GDP measures the monetary value of final goods and services—that is, those that are bought by the final
user—produced in a country in a given period of time (say a quarter or a year). It counts all of the output
generated within the borders of a country. GDP is composed of goods and services produced for sale in the
market and also includes some nonmarket production, such as defense or education services provided by
the government.
• Not all productive activity is included in GDP. For example, unpaid work (such as that performed in the
home or by volunteers) and black-market activities are not included because they are difficult to measure
and value accurately. That means, for example, that a baker who produces a loaf of bread for a customer
would contribute to GDP, but would not contribute to GDP if he baked the same loaf for his family (although
the ingredients he purchased would be counted). Hence, option (a) is the correct answer.

O
• It is also important to understand what GDP cannot tell us. GDP is not a measure of the overall standard
of living or well-being of a country. Although changes in the output off goods and services per person

C
(GDP per capita) are often used as a measure of whether the average citizen in a country is better or worse
off, it does not capture things that may be deemed important to general well-being.
S.
• So, for example, the increased output may come at the cost of environmental damage or other external costs
such as noise. Or it might involve the reduction of leisure time or the depletion of nonrenewable natural
TE

resources. The quality of life may also depend on the distribution of GDP among the residents of a country,
not just the overall level. To try to account for such factors, the United Nations computes a Human
Development Index, which ranks countries not only based on GDP per capita, but on other factors, such as
O

life expectancy, literacy, and school enrollment. Other attempts have been made to account for some of the
shortcomings of GDP, such as the Genuine Progress Indicator and the Gross National Happiness Index, but
FN

these too have their critics.


• The Least developed countries (LDCs) are low-income income countries confronting severe structural
impediments to sustainable development. They are highly vulnerable to economic and environmental
D

shocks and have low levels of human assets. The Committee for Development Policy (CDP) is mandated
.P

by the General Assembly (GA) and the Economic and Social Council (ECOSOC) to review the list of LDCs
every three years and to make recommendations on the inclusion and graduation of eligible countries. They
W

take Gross National Income per capita into accountt while determining the LDC status. .
W

28 www.visionias.in ©Vision IAS


W
.
• As an average measure, GDP also fails to capture wealth and income disparities within a society, often
negatively correlated with the health of that society. India's own experience tells us that GDP growth does
not always coincide with reduced income inequality. For the most recent decade, the top 10% group has
taken up more than 60% of the total wealth in India. This is in sharp contrast with the mere 6% of the total
wealth shared by the bottom 50% of the population, suggesting a significant increase of wealth inequality
in India over the past 40 years. In the advanced economies, Wilkinson and Pickett (2009), Atkinson
(2014) and Piketty (2020) show that higher inequality leads to adverse socio-economic outcomes but
income per capita, a measure of economic growth, has little impact.

Q 64.A
• Sea Slugs
o Why in news: Marine biologists and citizen scientists document unique species of sea slugs from the
Visakhapatnam coast.
o A unique species of nudibranch sea slugs were documented on the Visakhapatnam shore.
o These do not have shells and their gills are exposed.
o These sea slugs are found in places with abundant prey bases which may vary from sponges, hydroids,
and algae. Hence statement 1 is correct.
o Significance: The nudibranchs are usually found in coral reefs.
o Their presence is a significant indicator of a strong coral ecosystem. Hence statement 2 is correct.
o Shaggy Seahare was recorded for the first time from the coast.
o They mainly feed on algae and are usually seen in large numbers when there is an algal bloom.
o Photosynthetic Animals: 2 species of sap-eating sea slugs were also recorded that are the only
photosynthetic animals in the world. These can store chloroplast from the algae in their skin and do
photosynthesis in absence of food (Sap). Hence statement 3 is not correct.
o Most nudibranchs are active throughout the day, but there are some species that are also nocturnal in
nature.

Q 65.B
• The French attempted to establish a trade link with India as early as 1527. Taking a cue from the Portuguese
and the Dutch, the French commenced their commercial operations through the French East India Company,
established in 1664.
• Unlike other European powers which appeared in India through private trading companies, the
French commercial enterprise was a project of King Louis XIV. His minister of finance, Colbert, was
instrumental in establishing the French East India Company.
• As the French effort was a government initiative, it did not attract the general public of France who viewed
it as yet another way to tax people. The Company had been launched with a share capital of 5.5 million
livres out of which the monarch subscribed 3.5 million livres.
• Its directors were nominated by the king from the shareholders and they carried on the decisions of two
High Commissioners appointed by the Government. Hence statement 2 is correct.
• The French traders arrived in Madagascar (in Africa) in 1602. Though the French colonized Madagascar,

O
they had to abandon it in 1674, excepting a small coastal trading post. Berber, a French agent in India
obtained a firman (a royal command or authorization)on September 4, 1666 from Aurangzeb and the first

C
French factory was established at Surat in December 1668, much against the opposition of the Dutch.
Within a year the French established another factory at Masulipatnam. Hence statement 1 is not correct.
S.
• The French East India company lost to the English East India Company in the Carnatic Wars though
it did not lose its Indian presence forever. The Battle of Chinsura was as fought between the British East
TE

India Company and Mir Jafar, the Nawab of Bengal helped by Dutch. Hence statement 3 is not
correct.
O

Q 66.B
• The Universal Service Obligation Fund (USOF) is a fund under the Department of
FN

Telecommunications, Ministry of Communication. Hence statement 2 is not correct.


• It aims to provide a balance between the provision of Universal Service to all uncovered areas, including
the rural areas. Hence statement 1 is correct
D

• The USOF is generated by the Universal Service Levy (USL) which is collected from the Service
.P

Providers at a defined percentage of Adjusted Gross Revenue (AGR) as a component of the Licence
Fee. Such collections are credited
redited to the Consolidated fund of India.
India. The allocation of funds from the
W

USOF needs Parliamentary ary approval (due to the presence of the Consolidated Fund of India). Hence
statement 3 is correct.
W

29 www.visionias.in ©Vision IAS


W
.
• In fact, the collections for the USOF do not lapse at the end of the financial year.
• The New Telecom Policy (NTP), 1999 of the Department of Telecom, GoI had Universal Service as one
of its main objectives. Further, it envisaged the implementation of USOF for rural and remote areas would
be undertaken by all fixed service providers who shall be reimbursed from the USOF. The Indian
Telegraph (Amendment) Act, 2003 gave statutory status to USOF.

Q 67.A
• GSAT 7 series satellites are advanced satellites developed by ISRO to meet the communication needs
of the defence services. The GSAT 7 provides a gamut of services for military communication needs, which
includes low bit voice rate to high bit-rate data facilities, including multi-band communications. Hence,
statement 1 is correct and statement 2 is not correct.
• GSAT 7 satellite was launched in August 2013 from an Ariane 5 ECA rocket from Kourou in French
Guiana. It is a 2,650 kg satellite which has a footprint of nearly 2,000 nautical miles in the Indian Ocean
region.
• GSAT 7A (launched in 2018) helps in boosting the connectivity between the ground radar stations, airbases
and the airborne early warning and control aircraft (AEW&C) of the IAF (Indian Air Force).
• The proposal to launch GSAT 7B was cleared recently by the Defence Acquisition Council. GSAT 7B will
primarily fulfil the communication needs of the Army. Currently, the Army is using 30 per cent of the
communication capabilities of the GSAT 7A satellite, which has been designed for the Indian Air Force
(IAF).

Q 68.D
• The right of an institution, whether run by a majority or minority community, to get government aid
is not a fundamental right. Both have to equally follow the rules and conditions of the aid, the Supreme
Court held. Whether it is an institution run by the majority or the minority, all conditions that have relevance
to the proper utilization of the grant-in-aid by an educational institution can be imposed.
• All that Article 30(2) states is that on the ground that an institution is under the management of a minority,
whether based on religion or language, the grant of aid to that educational institution cannot be discriminated
against if other educational institutions are entitled to receive aid,” a Bench of Justices S.K. Kaul and M.M.
Sundresh said referring to the court’s historic T.M.A Pai decision in its judgment.
• Government aid to an institution is a matter of policy and it is not a fundamental right.
• A bench of Justices S K Kaul and M M Sundresh said grant of aid brings with it conditions which the
institution receiving it is bound to comply with. If an institution does not want to accept the conditions, it
can decline the grant but cannot say that the grant must be on its own terms, it said. A decision to grant aid
is by way of policy. While doing so, the government is not only concerned with the interest of the institutions
but the ability to undertake such an exercise.
• Hence, option (d) is the correct answer.

Q 69.A
• Recently, Centre has launched an online platform called Crime Multi Agency Centre (Cri-MAC) portal.

O
• The Crime Multi Agency Centre (Cri-MAC) was launched in 2020 by the Ministry of Home Affairs
(MHA) to share information on crime and d criminals 24×7 with various law enforcement agencies and

C
ensure a seamless flow of information among them.
• The application run by the National Crime Records Bureau (NCRB) aims to help in early detection and
S.
prevention of crime incidents across the country.
• Cri-MAC
MAC facilitates dissemination of information about significant crimes, including human trafficking
TE

across the country on real-time basis and enables inter-State coordination. This can help in locating and
identifying the trafficked victims as also in
n prevention, detection and investigation of crime.
• Hence option (a) is the correct answer.
O

Q 70.D
FN

• Metal Injection Molding is the manufacturing of solid metal parts utilizing injection molding
technology. It is also known as metal injection molding (MIM) and powder injection molding (PIM).
• In metal injection molding, finely powdered metal is mixed with a plastic binder to produce a feedstock
D

suitable for injection molding. The powdered metal and binder mixture is melted, shaped, and solidified in
.P

a standard
rd molding machine. The molded parts are then subjected to binder removal and sintering processes
post-molding
to remove the plastic binders and increase the density or compactness of the metal parts. These post
W

processes also ensure the metal part’s right geometry,


etry, composition, and physical properties.
W

30 www.visionias.in ©Vision IAS


W
.
• When it comes to fabricating high-volume parts and small, complex parts, MIM is more cost-efficient and
a lot less time-consuming than most other metal fabrication methods. Both production time and finishing
time are relatively short, which are just some of the reasons why the process is such a popular one
today.
• Some popular products that are made using metal injection molding include:
o Computer hinges for laptops
o Watch cases
o Plugs for cell phones
o Products for industries such as medical/dental, aerospace, firearms, automotive, commercial,
industrial, and more
• Hence, option (d) is the correct answer.

Q 71.D
• In 1991, India met with an economic crisis relating to its external debt — the government was not able to
make repayments on its borrowings from abroad; foreign exchange reserves, which we generally maintain
to import petrol and other important items, dropped to levels that were not sufficient for even a fortnight.
The crisis was further compounded by rising prices of essential goods. All these led the government to
introduce a new set of policy measures. India agreed to the conditionalities of the World Bank and IMF and
announced the New Economic Policy (NEP). Some of the reforms undertaken include:
o Tax reforms are concerned with the reforms in the government’s taxation and public expenditure
policies which are collectively known as its fiscal policy. Since 1991, there has been a continuous
reduction in the taxes on individual incomes as it was felt that high rates of income tax were an
important reason for tax evasion. It is now widely accepted that moderate rates of income tax
encourage savings and voluntary disclosure of income. The rate of corporation tax, which was very
high earlier, has been gradually reduced. Hence option 1 is not correct.
o The reform policies led to the establishment of private sector banks, Indian as well as
foreign. Foreign investment limit in banks was raised to around 50 per cent. Those banks which fulfil
certain conditions have been given the freedom to set up new branches without the approval of the RBI
and rationalise their existing branch networks. Hence option 2 is correct.
o The first important reform in the external sector was made in the foreign exchange market. In 1991, as
an immediate measure to resolve the balance of payments crisis, the rupee was devalued against foreign
currencies. This led to an increase in the inflow of foreign exchange. It also set the tone to free the
determination of rupee value in the foreign exchange market from government control. Now, more often
than not, markets determine exchange rates based on the demand and supply of foreign
exchange. Hence option 3 is correct.
o A major component of the financial sector reform process was the deregulation of a complex
structure of deposit and lending interest rates. The administered interest rate structure proved to be
inefficient. It, therefore, became necessary to reform the interest rate structure. Deregulation of interest
rates was intended to strengthen the competitive forces, improve allocative efficiency of resources and
strengthen the transmission of monetary policy. Hence option 4 is correct.

O
Q 72.B

C
• Immune imprinting is a tendency of the body to repeat its immune response based on the first variant
it encountered through infection or vaccination when it comes across a newer or slightly different
S.
variant of the same pathogen.
• Over the years, scientists have realised that imprinting acts as a database for the immune system, helping it
TE

put up a better response to repeat infections. However, the problem occurs when a similar, not
identical, variant of the virus is encountered by the body. In such cases, the immune system, rather than
generating new B cells, activates memory B cells, which in turn produce “antibodies that bind to features
O

found in both the old and new strains, known as cross-reactive antibodies.
• Although these cross-reactive antibodies do offer some protection against the new strain, they aren’t as
FN

effective as the ones produced by the B cells when thee body first came across the original virus.
• Hence, option (b) is the correct answer.
D

Q 73.B
.P

• The
he term bhakti is defined as "devotion" or a passionate love for the Divine. Moksha or liberation from
rebirth was not achieved through the following of rules, regulations or societal ordering, but through simple
W

devotion to the Divine.


W

31 www.visionias.in ©Vision IAS


W
.
• Within the movement, historians of religion often classify bhakti traditions into two broad categories:
nirguna (without attributes) and saguna (with attributes). The former included traditions that focused
on the worship of an abstract form of god. On the other hand, saguna bhakti was the worship of specific
deities such as Shiva, Vishnu and his avatars (incarnations) and forms of the goddess or Devi, all often
conceptualised in anthropomorphic forms.
Nirgun Tradition
• Most of the monotheists belonged to the low castes and were aware that there existed unity in their ideas.
They were also aware of each other’s teachings and influence, and in their verses, they mention each other
and their predecessors in a manner suggesting ideological affinity among them.
• All of them were influenced by the Vaishnava concept of Bhakti, the Nathpanthi movement and Sufism,
reflecting a synthesis of these three traditions and projecting a nirguna orientation.
• They refused any formal association with the organized dominant religions of the time (Hinduism and
Islam) and criticized what they regarded as the negative aspects of these religions.
• Kabir was the earliest and undoubtedly the most powerful fibre of the Nirguna tradition or the
monotheistic movements. He belonged to a family of weavers and spent a greater part of his life in Banaras
(Kashi). He rejected the varnashrama and all conventions based on caste distinction and championed new
values, helping the emergence of new groups and new unorthodox or protestant sects.
• Among those who were influenced by Kabir was Baba Guru Nanak, who was a Khatri from Punjab.
The message of Baba Guru Nanak is spelt out in his hymns and teachings. These suggest that he
advocated a form of Nirguna bhakti.
• He firmly repudiated the external practices of the religions he saw around him. He rejected sacrifices, ritual
baths, image worship, austerities and the scriptures of both Hindus and Muslims. For Baba Guru Nanak, the
Absolute or "rab" had no gender or form. He proposed a simple way to connect to the Divine by
remembering and repeating the Divine Name, expressing his ideas through hymns called "shabad" in
Punjabi, the language of the region.
Saguna Tradition
• In the early 16 century, Vallabacharya, a popular bhakti saint, popularized the Krishna bhakti. Among those
who followed Vallabacharya’s footsteps were Surdas and Mirabai. Surdas popularized the Krishna cult in
north India.
• Mirabai is perhaps the best-known woman poet within the bhakti tradition. Her biographies have been
reconstructed primarily from the bhajans attributed to her, which were transmitted orally for centuries.
• According to these, she was a Rajput princess from Merta in Marwar who was married against her wishes
to a prince of the Sisodia clan of Mewar, Rajasthan. She defied her husband and did not submit to the
traditional role of wife and mother, instead recognising Krishna, the avatar of Vishnu, as her lover.
• Chaitanya played a major role in the establishment of Mathura as a bhakti centre. He professed that
the highest knowledge of all was the knowledge of devotion to Krishna, and the greatest of sorrows was
separation, or viraba from Krishna. Further, he taught that the longing that Radha and the Gopis had for
Krishna should be duplicated in the soul of every devotee in his search for oneness with Krishna.
• Hence option (b) is the correct answer.

O
C
Q 74.A
• In general terms, clearing is an arrangement of trade which is done through the exchange of obligations.
obligati ons.
S.
For the successful completion of trade clearing corporations is essential. Clearing Corporations are
commonly called as a clearinghouse or clearing firm and it is a corporation which is associated with a stock
TE

exchange, which is established to handle confirmation, settlement and delivery of transactions. Clearing
Corporations helps the stock exchanges and this stock exchanges, in turn, ensures that buying and selling
of securities are being correctly dealt with.
O

• Section 2(d) of the securities contract(regulation)(Stock


regulation)(Stock exchange and clearing corporation) regulations,
2018 defines clearing corporation as “ entity that is established to undertake the activity of clearing and
FN

settlement of trades in securities or other instruments or products that are dealt with or traded on a
recognized stock exchange and includes a clearinghouse. Hence statement 1 is correct.
• The European Union (EU) market regulator, European Securities and Market Authority (ESMA), wants
D

Indian regulators to sign an agreement that will givee it the power to monitor, supervise or audit Indian
.P

clearing corporations. In November 2022, ESMA de-recognised


recognised six Indian CCs due to “no co-operation
co
arrangements” between ESMA and the Indian regulators and set April 2023 deadline for the European banks
W

to stop doing business with them.


W

32 www.visionias.in ©Vision IAS


W
.
• The six CCs include Clearing Corporation of India (CCIL), supervised by RBI, Indian Clearing
Corporation, NSE Clearing, MCX Clearing Corporation, supervised by SEBI, and India
International Clearing Corporation and NSE IFSC Clearing Corporation, supervised by the IFSCA.
Between these six CCs, trades in India’s entire cash and derivatives market in equities, bonds and forex are
cleared and settled. Hence statement 2 is not correct.

Q 75.B
• A patent is an exclusive set of rights granted for an invention, which may be a product or process that
provides a new way of doing something or offers a new technical solution to a problem. Indian patents are
governed by the Indian Patent Act of 1970. Under the act, patents are granted if the invention fulfils the
following criteria:
o It should be novel
o It should have inventive step/s or it must be non-obvious
o It should be capable of Industrial application
o It should not attract the provisions of sections 3 and 4 of the Patents Act 1970
• The original Indian Patents Act did not grant patent protection to pharmaceutical products to ensure that
medicines were available to the masses at a low price. India has gradually aligned itself with international
regimes pertaining to intellectual property rights. It became a party to the Trade-Related Aspects of
Intellectual Property Rights (TRIPS) Agreement following its membership to the World Trade Organization
on January 1, 1995. Following this, it amended its internal patent laws to comply with TRIPS, most
notably in 2005, when it introduced pharmaceutical product patents into the legislation. Hence
statement 3 is not correct
• The Patents Act, 1970 has imposed certain ‘restrictions’ on patentability. A patent cannot be granted on
‘mere use of a known process, machine or apparatus unless such known process results in a new product or
employs at least one new reactant’. An important part of this act is Section 3(d). It does not allow
‘evergreening’ of patents to prevent innovator pharma companies from extending the patent beyond
the stipulated period of 20 years, to ensure that the monopoly does not extend forever. Hence statement
2 is correct.
• Recently, the Indian Patent Office struck down Johnson and Johnson’s latest evergreening attempt, ruling
that the methods it was trying to patent were commonly known in the pharma industry and were therefore
not patentable. Section 3(d) of the Patents Act states that salt forms and derivatives of known
substances are not patentable. The applicant cannot claim a patent on these methods and compositions of
salt forms that have been known in scientific world. Hence statement 1 is not correct.

Q 76.A
• Pandit Malviya played a big role in the Civil Disobedience and Non-cooperation movement which was led
by Mahatma Gandhi. On account of his services to the Congress he was elected its President in 1909,
1918, 1932, and 1933, but owing to his arrest by the Government of India, he could not preside over
the 1932 and 1933 sessions which had been banned. Thus he presided over the INC sessions twice.
Hence option 1 is correct.

O
• Motilal Nehru presided over the INC sessions of 1919 and 1928. In 1907, Motilal Nehru presided over
a provincial conference of Moderate politicians at Allahabad. In 1909 he was elected a member of the

C
United Provinces
rovinces Council. He attended the Delhi Durbar in 1911 in honor of the visit of King George V
and Queen Mary and became a member of the Allahabad Municipal Board and of the All India Congress
S.
Committee. He was elected President of the United Province Congress. He preside over the Amritsar
Congress in December 1919 and the Calcutta Congress in December 1928. Hence option 2 is correct.
TE

• In March 1931, Sardar Patel presided over the 46th session of the Indian National Congress which was
called upon to ratify the Gandhi - Irwin Pact, which had just then been concluded. Hence option 3 is not
correct.
O

Q 77.C
FN

• The Mohammedan Literacy Society of Calcutta was founded by Abdul Latif in 1863. Hence statement
1 is correct.
• It was one of the earliest organizations that promoted modern education among upper and middle middle-class
D

Muslims. The goal of the society was the education of Muslim youth in English medium schools that would
.P

allow them to compete with their English and Hindu peers. Hence statement 2 is correct.
• It also played an important role in promoting Hindu-Muslim
Muslim unity.
W

• Abdul Latif himself was its secretary and his residence at 16 Taltala (Calcutta) was its head office.
W

33 www.visionias.in ©Vision IAS


W
.
• The management committee of the society was formed with Prince Mahomed Ruheemoodin of
Mysore as President, Prince Mirza Jahan Kader Bahadur of Oudh, and Prince Mahomed Nusseroodin
Hyder of Mysore as Vice-Presidents.

Q 78.D
• Recent context: CITES COP19 urged parties to remove references to pangolins from pharmacopoeia-an
official collection of approved pharmaceutical standards.
• Out of the eight species of pangolin, the Indian Pangolin (Manis crassicaudata) and the Chinese
Pangolin (Manis pentadactyla) are found in India.
• Out of eight species, population of four Asian pangolin species, including Indian Pangolin is reported to
have declined significantly in many areas due to hunting and trade.
• Indian Pangolin is a large anteater covered by 11-13 rows of scales on the back. A terminal scale is also
present on the lower side of the tail of the Indian Pangolin, which is absent in the Chinese Pangolin.
• Habitat
o Indian Pangolin is widely distributed in India, except the arid region, high Himalayas and the North-
East. The species is also found in Bangladesh, Pakistan, Nepal and Sri Lanka.
o Chinese Pangolin is found in the Himalayan foothills in Eastern Nepal, Bhutan, Northern India, North-
East Bangladesh and through Southern China.
• Threats to Pangolins in India: Hunting and poaching for local consumptive use (e.g. as a protein source and
traditional medicine) and international trade for its meat and scales in East and South East Asian countries,
particularly China and Vietnam.
• Protection Status:
o IUCN Red List:
✓ Indian Pangolin: Endangered
✓ Chinese Pangolin: Critically Endangered
o Both these species are listed under Schedule I of the Wildlife (Protection) Act, 1972. Hence option (d)
is the correct answer.

Q 79.A
• Project Greensand
o Why in news: Project Greensand will be the world’s first cross-border carbon storage project.
o Project Greensand is a carbon storage project in Denmark's North Sea. Hence option (a) is the
correct answer.
o The project aims to bury vast amounts of planet-heating carbon dioxide gas beneath the North Sea floor.
o It hopes that it can help the Nordic nation and others meet climate targets.
o It reverses the traffic in the pipes by pumping CO2 into the depleted reservoir of the Nini West oil field
in Denmark.
o Carbon Capture Storage (CCS):
✓ Capture and storage of CO2 go under the name CCS. CO2 is captured and stored underground.
✓ The UN climate panel points out that CCS has the potential to contribute significantly to achieving

O
the goal of reducing CO2 emissions.

C
Q 80.B
• Recent context: At the end of January 2023, India and the U.S. launched a programme to enhance
S.
their strategic partnership with delegations
egations led by the National Security Advisor (NSA) Ajit Doval
and his American counterpart, Jake Sullivan, meeting in Washington for the inaugural dialogue of
TE

the Initiative on Critical and Emerging Technologies (iCET).


• The initiative is a particularly significant
gnificant milestone in the bilateral relationship, having been announced at
the highest level — by Indian Prime Minister Narendra Modi and U.S. President Joe Biden in May
O

2022. The two leaders met on the margins of the QUAD leaders’ summit in Tokyo, Japan. Hence
statement 1 is not correct.
FN

• It is being run by the National Security Councils with the meetings by the National Security Advisors
of both the countries. Hence statement 2 is correct.
• The iCET initiative was launched with the goal "to elevate and expand" expand" Indo-U.S.
Indo strategic
D

technology partnership and defence industrial cooperation between the governments, businesses, and
.P

academic institutions of the two countries.


• There are six areas of cooperation under it: scientific research and development; quant quantum and artificial
W

intelligence, defense innovation, space, advanced telecom which would include things like 6G and
semiconductors.
W

34 www.visionias.in ©Vision IAS


W
.
Q 81.C
• The study titled “Report on municipal finances” reveals how municipal bodies are increasingly dependent
on fund transfers from the State and the Centre, while their revenue-earning capacity is limited. Their
revenue-raising powers are curtailed, the study shows. The combined budget of all the municipal
corporations in India is much smaller than that of the Central and State governments according to
the RBI analysis of the finances of urban local bodies.
• Limited funds aside, about 70% of it gets spent on salaries, pensions, and administrative expenses with the
rest left for capital expenditure. And above all, municipal corporations don’t borrow much, leaving them
gasping for funds.
• Taxes earned by municipal corporations in India are grossly inadequate to meet their expenditure needs. In
India, the own tax revenue of municipal corporations, comprising property tax, water tax, toll tax,
and other local taxes, formed 31-34% of the total revenue in the FY18-FY20 period. This share was
low compared to many other countries and it also declined over time.
• Using budgetary data from 201 municipal corporations across India, the RBI report calculated their overall
revenue receipts — consisting of their own tax revenue, own non-tax revenue, and transfers. In 2017-18
(actuals), it was estimated to be 0.61% of the GDP and according to budget estimates for 2019-20, it
increased slightly to 0.72% of the GDP. This was much smaller than Brazil’s 7% and South Africa’s 6%.
• Large variations can be observed if the municipal corporations’ own tax revenue is sliced State-wise. The
own tax revenue of municipal corporations as a share of the State’s GDP in 2017-18 crossed the 1% mark
in Delhi, Gujarat, Chandigarh, Maharashtra, and Chhattisgarh, while it was 0.1% or less in Karnataka, Goa,
Assam, and Sikkim.
• Another major issue with the municipal corporations’ revenue-raising capabilities was their dependence on
property taxes. In 2017-18, property taxes formed over 40% of the municipal corporations’ own tax revenue.
Despite such dominance, property tax collection in India was much lower compared to OECD countries
due to undervaluation, and poor administration, the report argues.
• The shortage of tax collectors has further impacted the revenues. The corporations are mostly dependent on
transfers with their revenue-raising potential being limited. Property taxes are not efficiently collected. The
generated funds are mostly spent on revenue expenditure, leaving a much smaller pie for capacity building.
• Municipal corporations need to explore different innovative bond and land-based financing mechanisms to
augment their resources. In India, revenues of Municipal Corporations (MCs) are dominated by property
tax collections and devolution of taxes and grants from upper tiers of government. MCs mostly rely on
borrowings from banks and financial institutions and loans from Centre/State governments to
finance their resource gaps in the absence of a well-developed market for municipal bonds,” it added.
• Hence A is true but R is false.

Q 82.D
• Cloud Computing is a method of storing and processing data on remote servers rather than locally.
Thus, servers are located away from the sources of data generation. Hence, statement 1 is correct.
• Cloud storage (cloud computing) is classified as decentralized if the storage system is distributed
around the world, and centralized if the storage system is singularly controlled. That means it can be

O
both centralized and decentralized. In the centralized version, the storage system is maintained by
the cloud controller, and it is operated by the central server in the cloud controller. Cloud computing
ng

C
allows users to access their files from anywhere at any time, but it also means that they cannot control
what happens with their data once it has been uploaded to servers owned by another company or
S.
organization. Hence, statement 2 is correct.
• Cloud computing
puting focuses on storing and processing large amounts of unstructured data at one time. Data
TE

processing happens in the cloud. Cloud computing lends itself more naturally to applications where
large amounts of data need to be processed at once (such as image recognition). Hence, statement 3 is
correct.
O

Q 83.B
FN

• Biotransformation technology is a novel approach that can alter the state of plastics and make them
biodegradable without leaving behind any microplastics. The tech was co-developed
co-developed
co-developed by the Imperial
College in London, UK, and a Britain-based
based startup, Polymateria.
D

• Plastics made using this technology are given a pre-programmed


programmed time during which the manufactured
.P

material looks and feels like conventional plastics without compromising on quality. Once the p product
expires and is exposed to the external environment, it self-destructs
destructs and biotransforms into bioavailable
W

wax. This wax is then consumed by microorganisms, converting waste into water, CO2, and biomass.
• Hence, option (b) is the correct answer.
W

35 www.visionias.in ©Vision IAS


W
.
Q 84.A
• Consumer Food Price Index (CFPI) is a measure of change in retail prices of food items consumed by the
population.
• It is released by National Statistics Office (NSO). Hence statement 1 is correct.
• It is released in three categories: rural, urban and combined- on all India basis with 2012 as base
year. Hence statements 2 and 3 are not correct.
• CFPI is based on retail price quotations as in the case of CPI, whereas the food index of WPI is obtained
from wholesale price quotations
• Like Consumer Price Index (CPI), the CFPI is also calculated on a monthly basis and methodology remains
the same as CPI.
• In CPI, food and beverage group constitutes 45.86% of weight. Out of this, beverages and some of the
eatable items are excluded to derive the CFPI basket of commodities. Weight of the CFPI commodities is
39.05% of the entire CPI basket.
• Additional information
• WPI is based on the share of the respective items in total whole-sale transactions in the economy at first
point of sale.
o Released By: Office of the Economic Adviser, Department for Promotion of Industry and Internal
Trade, Ministry of Commerce and Industries.
• CPI also known as retail inflation, CPI-based inflation reflects the price movements at the retail level.
o Released by: National Statistical Office (NSO), under the Ministry of Statistics and Programme
Implementation.
Q 85.D
• Bioindicators are species that show a response to problems in an ecosystem such as chemicals,
pesticides, and pollution. Bioindicators may display deformities or die due to the changes in their
environment, alerting scientist that action needs to be taken.
• Dragonflies are an excellent bio-indicator of water health. Their entire lifespan is intricately tied to the
water which means they need high water quality to survive to adulthood.
• Lichens are powerful Bioindicators of air quality. They are sensitive to atmospheric pollution such as
nitrogen (N) because they receive all their nutrients and water from wet and dry atmospheric deposition
(fall out).
• Algae growth is an indicator of water contaminants. Increase in mineral content of water due to agricultural
chemicals and fertilizers, which are rich in nitrogen and phosphorus, lead to the rapid growth of algae in the
water body.
• Frogs are good bioindicators as they show a sensitivity to many chemicals in the environment. Some of the
chemicals that enter the waterways cause changes in the frog’s DNA or RNA.
• Hence option (d) is the correct answer.

Q 86.A
• Recent context: ICMR advises against indiscriminate antibiotic use amid rising cases linked to influenza
Subtype A H3N2.

O
• Influenza viruses belong to Orthomyxoviridae family of viruses. Its nucleic acid consists of single
stranded RNA. Hence statement 1 is correct.

C
• Seasonal influenza is an acute respiratory infection caused by influenza viruses which circulate in all parts
of the world.
S.
• There are 4 types of seasonal influenza viruses, types A, B, C and D. Influenza A and B viruses circulate
and cause seasonal epidemics of disease.
TE

o Influenza A viruses are further classified into subtypes according to the combinations of the
hemagglutinin (HA) and the neuraminidase (NA), the proteins on the surface of the virus. Currently
circulating in humans are subtype A(H1N1) and A(H3N2) influenza viruses. Only influenza type
O

A viruses are known to have caused pandemics.


o Influenza B viruses are not classified into subtypes, but can be broken down into lineages. Currently
FN

circulating influenza type B viruses belong to either B/Yamagataa or B/Victoria lineage.


o Influenza C virus is detected less frequently and usually causes mild infections, thus does not present
public health importance.
D

o Influenza D viruses primarily affect cattle and are not known to infect or cause illness in people. Hence
.P

statement 2 is not correct.


• How can a person catch a flu virus from a pig?
W

o Influenza viruses can spread from pigs to people and from people to pigs. Spread from infected
pigs to humans is thought to happen in the same way that seasonal influenza viru
viruses spread between
W

36 www.visionias.in ©Vision IAS


W
.
people; mainly through infected droplets created when an infected pig coughs or sneezes. If these
droplets land in your nose or mouth, or you inhale them, you can be infected. Hence statement 3 is
correct.
• Signs and symptoms: Influenza is characterized by a sudden, rapid onset of symptoms. Influenza
symptoms may include fever, chills, body aches, sore throat, non-productive cough, runny nose and
headache. Gastrointestinal symptoms and muscle inflammation occur more often in young children, and
infants can present with a sepsis-like syndrome.
• Mode of Transmission: The transmission is air borne from person-to-person, through large droplets
generated by the act coughing and sneezing. These droplets when inhaled are highly contagious to
susceptible persons.
• Incubation period: Incubation period is 1- 4 days (typically 2-3 days). Viral shedding can begin before
symptom onset and peaks on day 1 of the symptoms. Adults may continue to shed virus for 4-6 days,
Children and Immuno suppressed/immune- compromised patients affected with influenza can shed for
months.
• Treatment:
o Antiviral drugs need to be taken within 2 days of your first flu symptoms. These drugs can make you
better faster and may also prevent serious complications.
o When you have flu, antibiotics will not help you feel better. Antibiotics won’t help you, and their side
effects could cause harm.

Q 87.A
• Nominal Exchange Rate: The nominal exchange rate E is defined as the number of units of the domestic
currency that can purchase a unit of a given foreign currency. A decrease in this variable is termed nominal
appreciation of the currency. An increase in this variable is termed the nominal depreciation of the currency.
The nominal exchange rate measures the current value of a currency against another. For e.g. it costs a rupee
holder 75 rupees to buy one US dollar.
• Real Exchange Rate: The real exchange rate R is defined as the ratio of the price level abroad and the
domestic price level, where the foreign price level is converted into domestic currency units via the current
nominal exchange rate. Formally, R=(E.P*)/P, where the foreign price level is denoted as P* and the
domestic price level as P. A decrease in R is termed appreciation of the real exchange rate, an increase is
termed depreciation.
• The real rate tells us how many times more or less goods and services can be purchased abroad (after
conversion into a foreign currency) than in the domestic market for a given amount. Rather than focusing
on the nominal exchange rate, it is more sensible to monitor the real exchange rate when assessing the
effect of exchange rates on the international trade or export competitiveness of a country. Hence,
statement 1 is correct.
• Purchasing Power Parity (PPP) Exchange Rate—the rate at which the currency of one country would
have to be converted into that of another country to buy the same amount of goods and services in each
country.
o When Real Exchange Rate = 1, Nominal Exchange Rate = PPP Exchange rate, we say that the currencies

O
are at purchasing power parity. This means that goods cost the same in two countries when measured
in the same currency. Hence, statement 2 is not correct.

C
o The purchasing power parity between two countries is the rate at which the currency of one country
needs to be converted into that of a second country to ensure that a given amount of the first country's
S.
currency will purchase the same volume of goods and services in the second country as it does in the
first.
TE

Q 88.B
• PM-Mudra yojana (PMMY) launched to create an inclusive, sustainable and value based entrepreneurial
O

culture. MUDRA provides refinance support to micro business under the Scheme of Pradhan Mantri
MUDRA Yojana. MUDRA does not lend directly to the micro entrepreneurs / individuals, these loans
FN

are given by Commercial Banks, RRBs, Small Finance Banks, MFIs and NBFCs. Hence statement 1
is correct.
• It provides collateral-free loans up to 10 lakh to the non-corporate,
corporate, non-farm
non-farm
non- farm small/micro enterprises.
D

These loans are classified as MUDRA loans under PMMY. Hence statement 2 is not correct.
.P

• Under the aegis of PMMY, MUDRA loans are classified into 3 categories namely 'Shishu', 'Kishore' and
'Tarun' to signify the stage of growth / development and funding needs of the beneficiary micro unit /
W

entrepreneur and also provide a reference point


oint for the next phase of graduation / growth. Hence statement
3 is correct.
W

37 www.visionias.in ©Vision IAS


W
.
o Shishu: covering loans upto Rs. 50,000
o Kishor: covering loans above Rs.50,000/- and upto Rs. 5 lakh
o Tarun: covering loans above Rs. 5 lakh to Rs. 10 lakh

Q 89.B
• Ujjain is an ancient and historical city that is 5000 years old. It was also called as Avantika. It is described
as the best city in Adi Brahma Purana and it is called mokshda and bhukti-mukti in Agni Purana and Garuda
Purana. There was a time when this city had been a capital of a big empire.
• Great scholars of varied fields like Kalidas, Varahmihir, Banabhatta, Rajashekera, Pushpadanta,
Shankaracharya, Vallabhacharya, Bhartahari, Diwakar, Kattayayan and Bhas had their associations
with Ujjain.
• Mughal emperor Akbar made this city his regional capital.
• Marathas ruled over here prior to the 18th century. Sindhiya Dynasty rulers worked for the promotion of
the Hindu religion.
• In 1235, Iltutmish invaded and plundered this city.
• King Vikramaditya made this city his capital the great scholar of Sanskrit Kalidas was in this court.
In the year 1810, Sindhias shifted their capital from Ujjain to Gwalior. In this city itself, king Bhartari
took the “Vairaagya Diksha”. In the Naatha Tradition of religious sect through his teacher Guru
Gorakshnaath. For centuries, Ujjain has been a center of religion for the Hindu, Jain and Buddha religions.
• In Ujjain, the Kumbh Mela (Simhastha Kumbh) is held every 12 years when the zodiac position of
Jupiter is in Leo.
• The spiritual city is famous for the Mahakal temple where lies one of the 12 Jyotirlingas in the country.
• Hence, option (b) is the correct answer.

Q 90.C
• Fluoride Remediation: A new study shows rice husk biochar removes fluoride from groundwater and
reduces fluorosis. Fluoride is one of the pollutants in groundwater.
• It is primarily caused by geological processes but is also contributed by anthropogenic activities across
India. A team from Nalanda University investigated the potential of renewable biochar produced from rice
husk biomass to absorb fluoride pollutants from groundwater.
• Fluoride remediation using biochars showed significant removal at neutral pH. Biochar-mediated sand
columns can be used for defluoridation in hand pumps and tube wells. Hence, statement 2 is correct.
o Fluorosis:
✓ It results from excessive fluoride intake from various sources, including food, water, air, and
excessive use of toothpaste.
✓ Prolonged fluoride consumption in higher concentrations in drinking water can cause dental
fluorosis, skeletal fluorosis, kidney diseases, and arthritis. Hence, statement 1 is correct.
✓ Dental fluorosis, or brown stains on the teeth caused by weakened enamel, is one of the most
noticeable symptoms due to fluorosis.

O
Q 91.A
• Recent context: Loitering munitions are now central to the United ted States (US) and the Russian militaries

C
for their respective theaters in the western Pacific and eastern Europe, according to its military leaders.
• A loitering munition is an aerial weapon system category in which the munition loiters around the target
S.
area for some time and attacks only once a target is located.
• It is also known as a suicide drone or kamikaze drone.
TE

• These are small unmanned aircraft that are packed with explosives and can be flown directly at a tank or a
group of troops, destroying them when they hit the target.
• Russia is accused of using Iranian-made
made drones in repeated attacks on Ukraine's civilian population during
O

the ongoing war in Ukraine.


• Hence, option (a) is the correct answer.
FN

Q 92.C
• The Indian Constitution establishes a judicial al system that is integrated as well as independent. The
D

Supreme Court stands at the top of the integrated judicial system in the country. Below it, there are high
.P

courts at the state level. Under a high court, there is a hierarchy of subordinate courts, tthat is, district courts
and other lower courts. This single system of courts enforces both the central laws as well as the state
W

laws, unlike in the USA, where the federal laws are enforced by the federal judiciary and the state
laws are enforced by the stateate judiciary. Hence statement 1 is correct.
W

38 www.visionias.in ©Vision IAS


W
.
• Constitution has established an independent judiciary. Neither legislature nor executive can intrude in its
domain. This has been done to ensure judicial independence. Integrated judiciary refers to the integrated
structure of the Indian judiciary with the Supreme Court at the top and the state high courts below it. While
the independent judicial system is a federal feature of the Constitution, the integrated judicial system
is a unitary feature. Hence statement 2 is correct.

Q 93.B
• The Morcha Chabian campaign for the recovery of the keys of the Golden Temple treasury, marked
a dramatic episode in the Sikh agitations in the early 1920s, to reform the management of their places
of worship. Hence, option (b) is the correct answer.
• The Golden Temple at Amritsar had been managed by a government-nominated sarbrdh (controller) since
1849. The Golden Temple came under Akali control in October 1920, but the Shiromani Gurdwara
Parbandhak Committee nominated the old sarbrdh, Sundar Singh Ramgarhia, as a member of the new
committee and appointed him to continue to administer the affairs of the Golden Temple. Even though the
sarbrdh now functioned under the directions of the Committee, but, since he still retained possession of the
keys of the Toshakhana (treasury) of the Golden Temple, some of the Akali reformers felt that
Governmental control, however nominal, still remained.
• On 11 November, the Government attempted to replace Sundar Singh with their own appointee Captain
Bahadur Singh, in effect overriding the choice of the SGPC. The Shiromani Gurdwara Parbandhak
Committee refused to recognize the new sarbrdh. On 12 November 1921, a protest meeting was convened
in Bagh Akaliari at Amritsar which was addressed by Baba Kharak Singh and other Akali leaders. Akali
meetings took place at Gujrariwala, Gujjar Khan and other places. Captain Bahadur Singh resigned, but the
Government remained adamant. Dan Singh of Vachhoa and Jasbal, two prominent Akalis, were arrested at
a divan at Ajnala on 26 November 1921.
• Failing to control the Sikh protests and foreseeing how it might affect Sikh soldiers and the peasantry, the
government announced on 3 January 1922 its decision to return the keys to the Shiromani Committee on
7/5 January 1922, but the Committee refused to accept the keys until all the Sikhs arrested during the
movement were released unconditionally.
• The matter was raised in the Punjab Legislative Council on 11 January 1922, Sir John Maynard, the Home
Member announced the release of all Sikhs under detention. However, the Akalis refused to go and fetch
the keys from the Deputy Commissioner. A Government official was eventually sent to deliver the keys
wrapped in a piece of red silk to Baba Kharak Singh, president of the Shiromani Gurdwara Parbandhak
Committee, at a divan on 19 January 1922 at the Akal Takht.
• In this agitation, the Sikh reformers were also joined by the Congress volunteers in Punjab. Since Mahatma
Gandhi's non-cooperation movement was already going on, the Punjab Government thought of isolating the
Akali reformers from the congress program of non-cooperation by releasing all the Akali volunteers arrested
in connection with the A agitation over the keys affair and returning the keys of the Golden temple treasury
to the President of the committee. This victory of the Akali reformers was seen by the national leaders as a
victory of the forces of nationalism.
Q 94.A

O
• Recently, the Supreme Court asked the Centre to provide data that may point to a more dignified, less
painful, and socially acceptable method of executing prisoners other than death by hanging.

C
• Section 354 (5) of the Code of Criminal Procedure mandates that a person sentenced to death shall “be
“be
hanged by the neck till he is dead”. Hence, statement 1 is correct.
S.
• While hanging by rope is the popular method of execution in India, Indian laws also allow a death-
death-row
death -row
row
convict to be shot dead by a firing squad. This is however allowed in limited circumstances and can be
TE

carried out only by the Army, the Navy, and the Air Force.
• As per the ‘Death Penalty in India: Annual Statistics 2022’ report released by Project 39A, at 100,
the State of Uttar Pradesh had the highest number of convicts on death row,, of which 32 sentences
O

were imposed in 2022. This was followed by Gujarat (61) and Jharkhand (46). Hence, statement 2 is not
correct.
FN

Q 95.A
• The greatest of the Vijayanagar rulers, Krishnadeva Raya, belonged to the Tuluva dynasty. Krishnadeva
D

Raya’s rule was characterised by expansion and consolidation.


.P

o Although the kingdom remained in a constant state of military preparedness, it flourished under
conditions of unparalleled peace and prosperity.
W

o Different languages such as Sanskrit, Telugu, Kannada and Tamil flourished in the regions. There was
a great development in Sanskritit and Telugu literature.
W

39 www.visionias.in ©Vision IAS


W
.
• Krishnadeva was a great patron of literature and art, and he was known as Andhra Bhoja. Eight eminent
scholars known as Ashtadiggajas were at his royal court.
o Allasani Peddanna was the greatest, and he was called Andhrakavita Pitamaga. His important
works include Manucharitam and Harikathasaram. Hence options 2 and 3 are not correct.
o Pingali Suranna and Tenali Ramakrishna were other important scholars.
• Krishnadeva Raya himself authored a Telugu work, Amukthamalyadha and Sanskrit works,
Jambavati Kalyanam and Ushaparinayam. Hence option 1 is correct.
• Amukthamalyadha: Considered a masterpiece in Telugu literature, the epic poem ‘Amukthamalyadha' tells
the well-known story of the daughter of Periazhvar, Goda Devi, also known as Andal, who used to wear the
garlands intended for Lord Ranganatha before they were offered to the deity, and hence the name ‘Amukta
Malya Da' — one who wears and gives away garlands.
o Sri Krishna Devaraya is believed to have written and dedicated the poem to Lord Venkateswara as
ordained by God in his dream.

Q 96.D
• Great Backyard Bird Count (GBBC) 2023
o It is an annual event that brings bird enthusiasts, students, and nature enthusiasts together for counting
birds they see around the places where they live, work, or study.
o Major Findings
✓ West Bengal reported the highest number of species (489 species) followed by Uttarakhand
(426), Arunachal Pradesh (407), Assam (397), and Karnataka (371) during the Great Backyard Bird
Count (GBBC) 2023. Tamil Nadu and Kerala took the eighth and ninth spots with 349 and 325
species.
✓ India’s birds are thriving in diverse habitats from the city to the countryside. A remarkable increase
in participation across the country helped India upload the second-highest number of checklists
after the United States of America and the third-highest species of any country.
o About GBBC
✓ The GBBC was launched in 1998 by the Cornell Lab of Ornithology and National Audubon
Society and was the first online citizen-science project (also referred to as community science) to
collect data on wild birds and to display results in near real-time. Bird Count India organizes the
GBBC in the country. Hence option (d) is the correct answer.
✓ In 2013, it became a global project after entering data into eBird, the world’s largest biodiversity-
related citizen science (community science) project.
Q 97.A
• As per Second Advance Estimates, the estimated production of major crops for 2022-23 is as under:
o Foodgrains – 3235.54 Lakh Tonnes
o Rice – 1308.37 Lakh Tonnes
o Wheat – 1121.82 Lakh Tonnes
o Nutri / Coarse Cereals – 527.26 Lakh Tonnes
o Maize – 346.13 Lakh Tonnes

O
o Barley – 22.04 Lakh Tonnes
o Total Pulses – 278.10 Lakh Tonnes

C
✓ Gram – 136.32 Lakh Tonnes
✓ Moong – 35.45 Lakh Tonnes
S.
✓ Gram has the dominant share among the pulses production in India.
o Oilseeds – 400.01 Lakh Tonnes
TE

o Groundnut – 100.56 Lakh Tonnes


o Soyabean – 139.75 Lakh Tonnes
o Rapeseed & Mustard – 128.18 Lakh Tonnes
O

o Cotton – 337.23 Lakh bales


o Sugarcane – 4687.89 Lakh Tonnes (468.7 million tonnes)
FN

✓ Hence statement 1 is correct.


o Jute & Mesta –100.49 Lakh bales
D
.P
W

o Kharif season oilseeds production is greater than Rabi season. Hence statement 2 is not correct.
W

40 www.visionias.in ©Vision IAS


W
.
• Sugarcane
o requires high temperatures, plenty of sunlight, large quantities of water (at least 1500 mm of rain per
year unless grown with irrigation), fertile soils, and good drainage.
o The crop cycle varies between 10 and 24 months but can be extended four times or more by additional
ratoon cropping.
Q 98.A
• Sunabeda Wildlife Sanctuary: Why in news: The strawberry harvest is done by farmers who live in one
of the 56 villages in the tropical deciduous forest of the Sunabeda Wildlife Sanctuary. The Sunabeda plateau
is largely a grass-covered plateau, along the Odisha-Chhattisgarh border
o The Sunabeda wildlife sanctuary is in Nuapada district, Odisha adjoining Udanti and Sitanadi
Sanctuaries of Chhattisgarh State.
o The sanctuary houses canyons and 11 waterfalls apart from diverse wildlife.
o The sanctuary forms the catchment area of the Jonk, Sunder, and Indra rivers.
o Fauna - Tigers, swamp deer, leopards, gaurs, sloth beer, barking deer, common langur, and rhesus
monkeys.
o Birds - The sanctuary is home to vultures, hill mynah, patridge, and peafowls.
o It forms a migration link for wild buffaloes across the two states of Odisha and Chhattisgarh.
o Chuktia Bhunjia tribe:
✓ The Chuktia Bhunjia tribes live inside the sanctuary. Hence option (a) is the correct answer,
✓ Chuktia Bhunjia tribe is one of 13 PVTGs found in Odisha.
✓ The Chuktia Bhunjia Development Agency (CBDA) works for the development of the tribe,
especially in livelihood programs.
Q 99.C
• C.R. Das served as the Indian National Congress president for a session and co-founded the Swaraj Party,
but almost over a century later, he is remembered as ‘Deshbandhu’, or ‘friend of the nation’, as the people
of his times affectionately called him. Das’ political career lasted only six years, but it came after he jumped
into India’s freedom struggle as a lawyer and helped many revolutionaries.
• Das completed his graduation from Presidency College in 1890. The same year, he went to England to
qualify for the ICS, which was dominated by the British. Unfortunately, he failed. As a result, he opted to
join the legal profession. Das practiced law at The Honourable Society of the Inner Temple in London in
England. While in England, Das campaigned for Dadabhai Naoroji to help win a seat in the House of
Commons from Central Finsbury. Naoroji became the first Asian in 1892 to become a part of Westminster.
Two years later, Das came back to India and started practising as a barrister at Calcutta High Court.
• As a lawyer, Das defended Indians who were accused of political offenses. He appeared in defence of
freedom fighters Brahmabandhab Upadhyay and Bhupendranath Datta who were charged with sedition.
• In 1908, Das attained widespread fame when he was successfully able to defend Aurobindo Ghose in
the Alipore Bomb Case — a bomb explosion in Muzaffarpur had killed two women and Ghose was
the main accused. Das, with meticulous handling of the case, concluded his address in the Calcutta
High Court with an impassioned appeal and got Ghose acquitted.
• He was a believer in the ‘Swadeshi’ idea and completely rejected the notion of development as promoted

O
by Western powers. In the Calcutta session, he put forward a plan for village reconstruction, which was to
entail steps such as the establishment of local self-government, and co-operative
operative credit societies as well as

C
re-starting the cottage industry.
• Das also brought out a newspaper called 'Forward' and later changed itss name to Liberty to fight the
S.
British Raj. Netaji Subhas Chandra Bose was the editor of this newspaper. C.R. was also a political
mentor of Subhas Chandra Bose. Hence, option (c) is the correct answer.
TE

Q 100.C
• Galaxy is a system of millions or billions off stars, together with gas and dust, held together by gravitational
O

attraction. They are the major building blocks of the universe.


• The smallest galaxies contain about 100,000 stars,ars, while the largest contains up to 3000 billion stars. From
FN

the billions of galaxies, two basic types have been identified:


o Regular galaxies, and
o Irregular galaxies.
D
.P
W
W

41 www.visionias.in ©Vision IAS


W
.
• Regular Galaxies:
o Spiral Galaxies:
✓ The Milky Way is an example of disc-shaped spiral galaxy which has a greater concentration of
stars near its centre.
✓ They consist of populations of old stars in the centre, and the youngest stars located in the arms.
✓ Spiral galaxies are well supplied with the interstellar gas in which new bright, young stars form.
✓ Smaller and less bright.
o Elliptical Galaxies:
✓ Star distribution is nonuniform.
✓ Most of their member stars are very old and no new star formation in them.
• Black dwarf:
o The last stage of stellar evolution is a black dwarf. Hence option (c) is the correct answer.
o A black dwarf is a white dwarf that has sufficiently cooled that it no longer emits significant heat or
light.
o Because the time required for a white dwarf to reach this state is calculated to be longer than the current
age of the universe (13.8 billion years), no black dwarfs are expected to exist in the universe yet.
• White dwarf:
o A white dwarf is very small, hot star, the last stage in the life cycle of a star like the Sun.
o White dwarfs are the remains of normal stars, whose nuclear energy supplies have been used up.
o White dwarf consists of degenerate matter with a very high density due to gravitational effects, i.e. one
spoonful has a mass of several tonnes.
• Although several dozen minor galaxies lie closer to our Milky Way the Andromeda galaxy is the nearest
large neighbour of our Milky Way. Excluding the Large and Small Magellanic Clouds, visible from
Earth’s Southern Hemisphere, the Andromeda galaxy is the brightest external galaxy you can see. At 2.5
million light-years, It’s the most distant thing most of us humans can see with the unaided eye.

O
C
S.
TE
O
FN
D
.P
W
W

42 www.visionias.in ©Vision IAS


W

You might also like